Ca oh h2so4: Найти окислительно — восстановительную реакцию в уравнениях: 1) Ca(OH)2 + H2SO4 → CaSO4 + 2H2O…

что, как сбалансировать и часто задаваемые вопросы —

By Дипали Арора

H2SO4 + Ca (OH)2 это реакция между сильной кислотой и сильным основанием. Давайте обсудим некоторые важные факты об этой реакции в этой статье.

H2SO4 является королем кислот и также широко известен как масло купороса. Он используется по-разному, например, для очистки других металлов и производства различных химикатов, красителей и т. Д. Ca (OH)2 образуется при смешивании CaO (негашеной извести) с водой. Он белый и в кристаллической форме.

Котлеты из фарша с …

Please enable JavaScript

Котлеты из фарша с грибами, удивительные вкусные котлеты!😋

С помощью полностью сбалансированного химического уравнения мы узнаем различные аспекты этой реакции, такие как энтальпия реакции, тип реакции и т. д. 

Что является произведением H

2SO4 и Са (ОН)2

Случай4 (сульфат кальция) и H2O (вода) — это два продукта, которые образуются, когда H2SO4 и Са (ОН)2 реагируют друг на друга.

Са (ОН)2 + H2SO4 → CaSO4 + 2ч2O

Какой тип реакции Н

2SO4 + Ca (OH)2

H2SO4 + Ca (OH)2 — это реакция нейтрализации в котором сильная кислота реагирует с сильным основанием с образованием соответствующей труднорастворимой соли.

Как сбалансировать H

2SO4 +Са(ОН)2
  • Напишите полное химическое уравнение скелета.
  • Напишите названия всех элементов, входящих в химическое уравнение.
ElementsКоличество атомов на стороне реагентаКоличество атомов на стороне продукта
Ca11
O65
H42
S11
Количество атомов
  • Уравняйте количество атомов как на стороне реагента, так и на стороне продукта. .

H

2SO4 + Ca (OH)2 титрование

Титрование H2SO4 с Са (ОН)2 в присутствии фенолфталеина является примером кислотно-щелочного титрования.

Используемый аппарат
  • Градуированный цилиндр
  • бюретка                                                                   
  • фенолфталеин
  • Мерная колба (помогает хранить раствор)                    
  • Подставка для колец
  • Маленькая воронка
  • Колба Эрленмейера (поле титрования)
Индикаторные

Индикатор, используемый в H2SO4 + CaSO4 Реакцией является фенолфталеин, потому что он легко меняет цвет при определенном рН.

ИндикаторныеЦвет в нормальном состоянииЦвет в базеЦвет в кислотах
фенолфталеинPinkрозовыйпрозрачный
Изменение окраски фенолфталеина
Процедура
  • Са (ОН)2 неизвестной молярности добавляют в колбу Эрленмейера.
  • Следующим шагом будет добавление 2 капель фенолфталеина.
  • Изменение цвета должно быть записано одновременно.
  • 3-молярная серная кислота разбавляется до низкой концентрации.
  • Бюретка используется для добавления нескольких капель к анализируемому веществу.
  • Раствор постоянно перемешивают.
  • Когда цвет начнет появляться, замедлите скорость капель, чтобы получить точный результат.
  • Когда цвет станет светло-розовым, запишите количество титрованный раствор используется для расчета концентрации вашего аналита
  • Для каждой другой концентрации серной кислоты повторите те же шаги.
Результат

Результаты титрования H2SO4 и Са (ОН)2 ненадежны, так как концентрации не остаются стабильными, поскольку внешние факторы вызывают нестабильность в испытаниях. Образующийся цвет не зависит от принятого раствора; скорее, это зависит от взятого индикатора, следовательно, образующийся цвет розовый.

H

2SO4 + Ca (OH)2 чистое ионное уравнение

Ионное уравнение реакции имеет вид,

Ca2+ + 2ОН + 2H+ SO42-Случай4(с) +2H2O(L)

Чистое ионное уравнение выводится с использованием следующих шагов:

  • Напишите полное сбалансированное уравнение.
  • Са (ОН)2 + H2SO4 → CaSO4 + 2ч2O
  • Добавьте вместе с ними физические состояния соединения.
  • Са (ОН)2(вод) + H2SO4(водн.) → CaSO4(водн.) + 2H2О (водн.)
  • Разбейте все соединения, растворимые в воде, на соответствующие ионы.
  • Ca2+ + 2ОН + 2H+ SO42-Случай4(с) +2H2O(L)
  • Вычеркните ионы, сходные как со стороны реагента, так и со стороны продукта.
  • Са (ОН)2(вод) + H2SO4(водн.) → CaSO4(водн.) + 2H2О (водн.)
  • Уравнение слева представляет собой результирующее ионное уравнение.  
  • Ca2+ + 2ОН + 2H+ SO42- →CaSO4(с) +2H2O(L)

H

2SO4 + Ca (OH)2 сопряженные пары
  • H2SO4 является сильной кислотой и ее сопряженным основанием является HSO4
  • Пара сопряженных кислот Ca (OH)2 это Са (ОН)+.

H

2SO4 и Са (ОН)2 межмолекулярные силы
  • Существует три типа межмолекулярных сил между молекулами H2SO4: дисперсионные силы Ван-дер-Ваальса, диполь-дипольные взаимодействия и водородная связь .
  • Са (ОН)2 представляет собой ионное соединение с сильными силами притяжения между его ионами.

H

2SO4 + Ca (OH)2 энтальпия реакции

 Реакция энтальпия из H2SO4 + Ca (OH)2 составляет -23.61 кДж.

Н

2SO4 + Ca (OH)2 буферный раствор

Реакция между H2SO4 и Са (ОН)2 не могу сделать буферный раствор потому что это реакция нейтрализации и полностью ионизируется.

Н

2SO4 + Ca (OH)2 полная реакция

H2SO4 и Са (ОН)2 являются полными реакциями, потому что реагенты взаимно обмениваются своими ионами с образованием двух продуктов.

Н

2SO4 + Ca (OH)2 экзотермическая или эндотермическая реакция

Реакция между H2SO4 и Са (ОН)2 очень экзотермический потому что энтальпия реакции очень высока; следовательно, выделяется много тепла.

Н

2SO4 + Ca (OH)2 окислительно-восстановительная реакция

Реакция между H2SO4 и Са (ОН)2 не является окислительно-восстановительной реакцией, поскольку степени окисления элементов не изменяются, что объясняется ниже..Степени окисления элементов

Н

2SO4 + Ca (OH)2 реакция осаждения

Это не реакция осадка, потому что осадок не образуется, когда H2SO4 соединяется с Ca (OH)2. Вместо этого он образует растворимую соль.

Н

2SO4 + Ca (OH)2 обратимая или необратимая реакция

Реакция между H2SO4 и Са (ОН)2 является обратимым, поскольку эта реакция имеет тенденцию двигаться как в прямом, так и в обратном направлении.

Н

2SO4 + Ca (OH)2 реакция смещения

Когда Н2SO4 соединяется с Ca (OH)2, двойное смещение происходит потому, что происходит взаимный обмен ионами. Ca соединяется с SO42- ионы и H+ сочетается с ОН ионы с образованием воды.

Заключение

Са (ОН)2 и Н2SO4 является кислотно-щелочной реакцией; следовательно, образующиеся продукты представляют собой соль и воду. Соль, CaSO4, вызывает постоянную жесткость воды и используется в качестве иссушать. Н2O имеет pH 7 и водородные связи между его молекулами.

Чему равен фактор эквивалентности гидроксида кальция в реакции h3SO4 +…

Начертите равнобедренный треугольник АВС с основанием АС и острым углом В.С помощью циркуля и линейки проведите высоту из вершины угла А

Сколькими нулями оканчивается произведение всех натуральных чисел от 41 до 64 включительно

Идеальный одноатомный газ находится в сосуде с жёсткими стенками объемом 0,6м(в кубе). При охлаждении его давление снизилось на 3*10(в 3ст)Па. На сколько уменьшилась внутренняя энергия газа? Ответ

Задача 1 Шар массой 2 кг движется со скоростью 4 м/с и сталкивается с неподвижным шаром массой 6 кг. Какова будет скорость и направление движения первого шара после упругого удара, если скорость

сокращенное ионное уравнение реакции Ba(2+) + SO4(2-) = BaSO4 соответствует взаимодействию

Пользуйтесь нашим приложением

Ответов пока нет

Михаил Александров

от 0 p.

Читать ответы

Андрей Андреевич

от 70 p.

Читать ответы

Eleonora Gabrielyan

от 0 p.

Читать ответы

Посмотреть всех экспертов из раздела Учеба и наука

Похожие вопросы

What, How to Balance & FAQs —

By Deepali Arora

H 2 SO 4 + Ca(OH) 2 представляет собой реакцию между сильной кислотой и сильным основанием. Давайте обсудим некоторые важные факты об этой реакции в этой статье.

H 2 SO 4 является королем кислот и также широко известен как купоросное масло. Он используется по-разному, например, для очистки других металлов и производства различных химикатов, красителей и т. д. Ca(OH) 2 получается при смешивании CaO (негашеной извести) с водой. Он белый и в кристаллической форме.

Конденсатор и емкость | Лучшие 2…

Пожалуйста, включите JavaScript

Конденсатор и емкость | Best 2022

С помощью полностью сбалансированного химического уравнения мы узнаем различные аспекты этой реакции, такие как энтальпия реакции, тип реакции и т. д. ) 2

CaSO 4 (сульфат кальция) и H 2 O (вода) — два продукта, которые образуются при взаимодействии H 2 SO 4 и Ca(OH) 2 .

CA (OH) 2 + H 2 SO 4 → CASO 4 + 2H 2 O

Какой тип реакции H

2 SO 4 + CA (CA (CA ( OH) 2

H 2 SO 4 + Ca(OH) 2 представляет собой реакцию нейтрализации, при которой сильная кислота реагирует с сильным основанием с образованием соответствующей труднорастворимой соли.

Как сбалансировать H

2 SO 4 +Ca(OH) 2
  • Напишите полное химическое уравнение скелета.
  • Напишите названия всех элементов, входящих в химическое уравнение.
Элементы Number of atoms on reactant side Number of atoms on product side
Ca 1 1
O 6 5
H 4 2
S 1 1
Количество атомов
  • число атомов на стороне реакции

H

2 SO 4 + CA (OH) 2 Титрование

Титрование H 2 SO 4 с CA (OH) 2 В присутствии фенолфталеина является примером кислоты. -основное титрование.

Устройство использовалось
  • градуированный цилиндр
  • Burette
  • Phenolphthalein
  • .0020
  • Volumetric Flask (helps in storing the solution)                    
  • Ring stand
  • Small funnel
  • Erlenmeyer Flask (Titration field)
Indicator

Индикатором, используемым в реакции H 2 SO 4 + CaSO 4 , является фенолфталеин, поскольку он легко меняет цвет при определенном значении pH.

Indicator Colour in normal condition Colour in bases Colour in acids
Phenolphthalein Pink pink transparent
Изменение цвета фенолфталеина

Процедура
  • Ca(OH) 2 неизвестной молярности добавляют в колбу Эрленмейера.
  • Следующим шагом будет добавление 2 капель фенолфталеина.
  • Изменение цвета должно быть записано одновременно.
  • 3 Молярная серная кислота разбавлена ​​до низкой концентрации.
  • Бюретка используется для добавления нескольких капель к анализируемому веществу.
  • Раствор постоянно перемешивается.
  • Когда цвет начнет проявляться, уменьшите скорость капель, чтобы получить точный результат.
  • Когда цвет станет светло-розовым, запишите количество титранта, используемого для расчета концентрации анализируемого вещества.
  • Для каждой другой концентрации серной кислоты повторите те же шаги.
Результат

Результаты титрования H 2 SO 4 и Ca(OH) 4 и Ca(OH) 2 являются причиной нестабильности внешних факторов, так как концентрации не остаются нестабильными. Образующийся цвет не зависит от принятого раствора; скорее, это зависит от взятого индикатора, следовательно, образующийся цвет розовый.

H

2 SO 4 + CA (OH) 2 Чистое ионное уравнение

Ионное уравнение для реакции составляет ,

CA 2+ + 2OH + 2H + 2H + 2H + 2H . + SO 4 2- CASO 4 (S) +2 H 2 O (L)

Уравнение чистого ионического

  • Напишите полное сбалансированное уравнение .
  • CA (OH) 2 + H 2 SO 4 → CASO 4 + 2H 2 O
    • Добавить физические состояния составной работы вместе с ними.
    • CA (OH) 2 (AQ) + H 2 SO 4 (AQ) → CASO 4 (AQ) + 2H 2 O (AQ)
    • BRECK BRECK). все соединения, растворимые в воде, на соответствующие ионы.
    • CA 2+ + 2OH + 2H + SO 4 2- CASO 4 (S) +2 H 2 6. 2020202020202020202020202020202020202020202.7202020202020202020202. 2020202020202020202020202. 20202020202020202020202. 2020202. (S) H 2 4 (S).
      • Вычеркните ионы, сходные как по реагенту, так и по продукту.
      • Ca(OH) 2 (aq) + H 2 SO 4 (aq) → CaSO 4 (aq) + 2H 2 O(aq)
      • The уравнение слева представляет собой результирующее ионное уравнение.
      • CA 2+ + 2OH + 2H + SO 4 2- → CASO 4 (S) +2 H → CASO 4 (S) +2 H .

      H

      2 SO 4 + Ca(OH) 2 conjugate pairs
      • H 2 SO 4 is a strong acid and its conjugate base is HSO 4
      • Пара сопряженных кислот Ca(OH) 2 представляет собой Ca(OH) + .

      H

      2 SO 4 и CA (OH) 2 Межмолекулярные силы
      • Существуют три типа межмолекулярных сил между молекулами H 2 SO 4 : Van Der Disersion Dispersionion itsionion itsionion itsionion haalsion haalsion desersion it waals it waals it waals desersion wals desersion waals waals waals waals waals waals waals waals waals waals waals waals силы, диполь-дипольные взаимодействия и водородные связи.
      • Ca(OH) 2 представляет собой ионное соединение с сильными силами притяжения между его ионами.

      H

      2 SO 4 + CA (OH) 2 Энтальпия реакции

      Реакционная энтальпия H 2 SO 4 + CA (OH) 2 —23,61 KJ.

      Is H

      2 SO 4 + Ca(OH) 2 буферный раствор

      раствор, потому что это реакция нейтрализации и полностью ионизируется.

      Is H

      2 SO 4 + Ca(OH) 2 полная реакция

      H 2 SO 4 2900 взаимно с образованием двух продуктов.

      IS H

      2 SO 4 + CA (OH) 2 Экзотермическая или эндотермическая реакция

      Реакция между H 2 SO 4 и CA (OH) 2 — очень экзотермическая потому что энтальпия реакции очень высока; следовательно, выделяется много тепла.

      Is H

      2 SO 4 + Ca(OH) 2 а окислительно-восстановительная реакция реакция, потому что нет изменения степени окисления элементов, что объясняется ниже
      . Степени окисления элементов

      Is H

      2 SO 4 + Ca(OH) 2 реакция осаждения

      Это не реакция осаждения, поскольку осадок не образуется, когда H 2 SO 4 соединяется с Ca(OH) 2 . Вместо этого он образует растворимую соль.

      IS H

      2 SO 4 + CA (OH) 2 Обратимая или необратимая реакция

      Реакция между H 2 SO 4 и CA (OH) 2 Обратима, потому что это 4 и CA (OH) 2 , потому что это реакция имеет тенденцию двигаться как в прямом, так и в обратном направлении.

      Is H

      2 SO 4 + Ca(OH) 2 реакция замещения

      Когда H 2 SO 4 соединяется с Ca(OH) 2 , происходит двойное смещение, поскольку происходит взаимный обмен ионами. Ca соединяется с ионами SO 4 2-, а H + соединяется с ионами OH с образованием воды.

      Заключение

      Ca(OH) 2 и H 2 SO 4 является кислотно-щелочной реакцией; следовательно, образующиеся продукты представляют собой соль и воду. Соль CaSO 4 вызывает постоянную жесткость воды и используется в качестве осушителя. Н 2 O имеет pH 7 и водородные связи между его молекулами.

      6.4: Классификация химических реакций (кислоты и основания) (задачи)

      1. Последнее обновление
      2. Сохранить как PDF
    • Идентификатор страницы
      106786
    • ЗАДАЧА \(\PageIndex{1}\)

      Заполните и сбалансируйте следующие кислотно-основные уравнения:

      1. Газ HCl реагирует с твердым Ca(OH) 2 ( s ).
      2. Раствор Sr(OH) 2 добавляют к раствору HNO 3 .
      Ответить на

      \(\ce{2HCl}(g)+\ce{Ca(OH)2}(s)\rightarrow \ce{CaCl2}(s)+\ce{2h3O}(l)\)

      Ответ б

      \(\ce{Sr(OH)2}(водн.)+\ce{2HNO3}(водн.)\rightarrow \ce{Sr(NO3)2}(водн.)+\ce{2h3O}(l)\)

      ЗАДАЧА \(\PageIndex{2}\)

      Дополните и сбалансируйте следующие кислотно-основные уравнения:

      1. Раствор HClO 4 добавляют к раствору LiOH.
      2. Водный H 2 SO 4 реагирует с NaOH.
      3. Ba(OH) 2 реагирует с газообразным HF.
      Ответ

      \(\ce{HClO4}(водн.)+\ce{LiOH}(водн.)\rightarrow \ce{LiClO4}(водн.)+\ce{h3O}(л)\)

      Ответ б

      \(\ce{h3SO4}(водн. )+\ce{2NaOH}(водн.)\rightarrow \ce{Na2SO4}(водн.)+\ce{2h3O}(л)\)

      Ответ c

      \(\ce{2HF}(водн.)+\ce{Ba(OH)2}(водн.)\rightarrow \ce{BaF2}(водн.)+\ce{2h3O}(l)\)

      Нажмите здесь, чтобы посмотреть видео о решении

      (Это видео имеет неправильное название, но является правильным решением проблемы )

      ЗАДАЧА \(\PageIndex{3}\)

      Дополните и сбалансируйте уравнения для следующих реакций кислотно-щелочной нейтрализации. Если в качестве растворителя используется вода, запишите реагенты и продукты как водные ионы. В некоторых случаях может быть более одного правильного ответа, в зависимости от количества используемых реагентов. 92+}(водн.)+\ce{2ClO4-}(водн. )+\ce{2h3O}(л)\)

      Ответ б

      \(\ce{SO3}(g)+\ce{2h3O}(l)\rightarrow \ce{h4O+}(водн.)+\ce{HSO4-}(водн.)\), (раствор H 2 СО 4 )

      Ответ c

      \(\ce{SrO}(s)+\ce{h3SO4}(l)\rightarrow \ce{SrSO4}(s)+\ce{h3O}\)

      ЗАДАЧА \(\PageIndex{4}\)

      Дополните и сбалансируйте уравнения следующих реакций, каждая из которых может быть использована для удаления сероводорода из природного газа:

      1. \(\ce{Ca(OH)2}(s)+\ce{h3S}(g) \rightarrow\)
      2. \(\ce{Na2CO3}(водн.)+\ce{h3S}(г)\стрелка вправо \)
      Ответить

      \(\ce{Ca(OH)2}(s)+\ce{h3S}(g)\rightarrow \ce{CaS}(s)+\ce{2h3O}(l)\)

      Ответить

      \(\ce{Na2CO3}(водн.)+\ce{h3S}(г)\rightarrow \ce{Na2S}(водн. )+\ce{CO2}(г)+\ce{h3O}(л)\)

      ПРОБЛЕМА \(\PageIndex{5}\)

      Цикламат кальция Ca(C 6 H 11 NHSO 3 ) 2 — искусственный подсластитель, используемый во многих странах мира, но запрещенный в США. Его можно очистить в промышленных масштабах путем превращения в соль бария путем реакции кислоты C 6 H 11 NHSO 3 H с карбонатом бария, обработкой серной кислотой (сульфат бария очень нерастворим) и последующей нейтрализацией с помощью гидроксид кальция. Напишите уравнения этих реакций.

      Ответить

      \(\ce{2C6h21NHSO3H}+\ce{BaCO3}\rightarrow \ce{Ba(C6h21NHSO3)2}+\ce{h3CO3}\)

      \(\ce{Ba(C6h21NHSO3)2}+\ce{h3SO4}\rightarrow \ce{BaSO4}+\ce{2C6h21NHSO3H}\)

      \(\ce{2C6h21NHSO3H}+\ce{Ca(OH)2}\rightarrow \ce{Ca(C6h21NHSO3)2}+\ce{2h3O}\)

       

      Авторы

      • Пол Флауэрс (Университет Северной Каролины, Пембрук), Клаус Теопольд (Университет Делавэра) и Ричард Лэнгли (Государственный университет Стивена Ф.

    Косинусы таблица: Таблица Брадиса косинусы

    Дистанционный репетитор — онлайн-репетиторы России и зарубежья

    КАК ПРОХОДЯТ
    ОНЛАЙН-ЗАНЯТИЯ?

    Ученик и учитель видят и слышат
    друг друга, совместно пишут на
    виртуальной доске, не выходя из
    дома!

    КАК ВЫБРАТЬ репетитора

    Выбрать репетитора самостоятельно

    ИЛИ

    Позвонить и Вам поможет специалист

    8 (800) 333 58 91

    * Звонок является бесплатным на территории РФ
    ** Время приема звонков с 10 до 22 по МСК

    ПОДАТЬ ЗАЯВКУ

    Россия +7Украина +380Австралия +61Белоруссия +375Великобритания +44Израиль +972Канада, США +1Китай +86Швейцария +41

    Выбранные репетиторы

    Заполните форму, и мы быстро и бесплатно подберем Вам дистанционного репетитора по Вашим пожеланиям.
    Менеджер свяжется с Вами в течение 15 минут и порекомендует специалиста.

    Отправляя форму, Вы принимаете Условия использования и даёте Согласие на обработку персональных данных

    Вы также можете воспользоваться
    расширенной формой подачи заявки

    Как оплачивать и СКОЛЬКО ЭТО СТОИТ

    от
    800 до 5000 ₽

    за 60 мин.

    и зависит

    ОТ ОПЫТА и
    квалификации
    репетитора

    ОТ ПОСТАВЛЕННЫХ ЦЕЛЕЙ ОБУЧЕНИЯ
    (например, подготовка к олимпиадам, ДВИ стоит дороже, чем подготовка к ЕГЭ)

    ОТ ПРЕДМЕТА (например, услуги репетиторовиностранных языков дороже)

    Оплата непосредственно репетитору, удобным для Вас способом

    Почему я выбираю DisTTutor

    БЫСТРЫЙ ПОДБОР
    РЕПЕТИТОРА И
    ИНДИВИДУАЛЬНЫЙ ПОДХОД

    ОПТИМАЛЬНОЕ
    СООТНОШЕНИЕ ЦЕНЫ И
    КАЧЕСТВА

    ПРОВЕРЕНЫ ДОКУМЕНТЫ ОБ ОБРАЗОВАНИИ У ВСЕХ РЕПЕТИТОРОВ

    НАДЕЖНОСТЬ И ОПЫТ.
    DisTTutor на рынке с 2008 года.

    ПРОВЕДЕНИЕ БЕСПЛАТНОГО, ПРОБНОГО УРОКА

    ЗАМЕНА РЕПЕТИТОРА, ЕСЛИ ЭТО НЕОБХОДИМО

    376313 УЧЕНИКОВ ИЗ РАЗНЫХ СТРАН МИРА
    уже сделали свой выбор

    И вот, что УЧЕНИКИ ГОВОРЯТ
    о наших репетиторах

    Владимир Александрович Кузьмин

    «

    Тренинг у Кузьмина В. А. проходил в экстремальных условиях. Мой модем совершенно не держал соединение. За время часового тренинга связь прерывалась практически постоянно. Ясно, что в таких условиях чрезвычайно непросто чему-то учить. Однако Владимир Александрович проявил удивительную выдержку и терпение. Неоднократно он перезванивал мне на сотовый телефон, чтобы дать пояснения или комментарии. Ценой больших усилий нам удалось рассмотреть три программы: ConceptDraw MINDMAP Professional Ru, GeoGebra и Ultra Flash Video FLV Converter. Владимир Александрович открыл мне курс на платформе dist-tutor.info и научил подключать и настраивать Виртуальный кабинет, порекомендовав изучать возможности этого ресурса, чтобы постепенно уходить от использования Skype. В итоге, занятие мне очень понравилось! Спокойное объяснение материала, дружелюбный настрой, подбадривание дистанционного ученика даже в самых непростых ситуациях — вот далеко не полный перечень качеств Владимира Александровича как дистанционного педагога. Мне следует учиться у такого замечательного репетитора!

    «

    Вячеслав Юрьевич Матыкин

    Чулпан Равилевна Насырова

    «

    Я очень довольна репетитором по химии. Очень хороший подход к ученику,внятно объясняет. У меня появились сдвиги, стала получать хорошие оценки по химии. Очень хороший преподаватель. Всем , кто хочет изучать химию, советую только её !!!

    «

    Алина Крякина

    Надежда Васильевна Токарева

    «

    Мы занимались с Надеждой Васильевной по математике 5 класса. Занятия проходили в удобное для обоих сторон время. Если необходимо было дополнительно позаниматься во внеурочное время, Надежда Васильевна всегда шла навстречу. Ей можно было позванить, чтобы просто задать вопрос по непонятной задачке из домашнего задания. Моя дочь существенно подняла свой уровень знаний по математике и начала демонстрировать хорошие оценки. Мы очень благодарны Надежде Васильевне за помощь в этом учебном году, надеемся на продолжение отношений осенью.

    «

    Эльмира Есеноманова

    Ольга Александровна Мухаметзянова

    «

    Подготовку к ЕГЭ по русскому языку мой сын начал с 10 класса. Ольга Александровна грамотный педагог, пунктуальный, ответственный человек. Она всегда старается построить занятие так, чтобы оно прошло максимально плодотворно и интересно. Нас абсолютно все устраивает в работе педагога. Сотрудничество приносит отличные результаты, и мы его продолжаем. Спасибо.

    «

    Оксана Александровна


    Клиентам

    • Репетиторы по математике
    • Репетиторы по русскому языку
    • Репетиторы по химии
    • Репетиторы по биологии
    • Репетиторы английского языка
    • Репетиторы немецкого языка

    Репетиторам

    • Регистрация
    • Публичная оферта
    • Библиотека
    • Бан-лист репетиторов

    Партнеры

    • ChemSchool
    • PREPY. RU
    • Class

    Колебания твердых тел

    Колебания твердых тел
      

    Колебания твердых тел, Ганиев Р. Ф., Кононенко В. О. Издательство «Наука», Главная редакция физико-математической литературы, М., 1976, — 432 с.

    Закономерности колебаний твердых тел являются основой для динамических расчетов и анализа движения, так как твердое тело служит расчетной моделью для многих машин, приборов, самолетов, космических аппаратов.

    В книге излагается теория колебаний твердых тел. Основное внимание уделяется нелинейным колебаниям в пространственном движении, поскольку актуальность этих вопросов возрастает с развитием авиационной и космической техники, скоростного наземного транспорта, навигационных приборов. Детально исследуются возможности и условия возникновения нелинейных резонансных колебаний. Анализируются периодические и почти-периодические режимы резонансных колебаний и их устойчивость.

    Книга рассчитана на научных работников, инженеров и студентов, занимающихся вопросами динамики механических систем в разнообразных областях техники.



    Оглавление

    ПРЕДИСЛОВИЕ
    ВВЕДЕНИЕ
    ГЛАВА I. ЭЛЕМЕНТЫ КИНЕМАТИКИ И ДИНАМИКИ ТВЕРДОГО ТЕЛА
    § 1. Элементы кинематики твердого тела
    § 2. Динамические величины
    § 3. Дифференциальные уравнения движения твердого тела
    § 4. Лагранжевы дифференциальные уравнения движения твердого тела
    ГЛАВА II. УРАВНЕНИЯ КОЛЕБАНИЙ ТВЕРДОГО ТЕЛА
    § 1. Уравнения движения твердого тела в потенциальном поле сил
    § 2. Линеаризованные уравнения движения твердого тела
    § 3. Квазилинейные уравнения колебаний твердого тела
    ГЛАВА III. ЛИНЕЙНЫЕ И БЛИЗКИЕ К НИМ КОЛЕБАНИЯ ТВЕРДОГО ТЕЛА
    § 1. Собственные колебания твердого тела
    § 2. Возможности изолированных колебаний в направлениях обобщенных координат
    § 3. Вынужденные колебания твердого тела
    § 4. Свободные колебания квазилинейного типа
    § 5. Вынужденные нерезонансные колебания
    ГЛАВА IV. НЕУСТОЙЧИВОСТЬ ПЕРИОДИЧЕСКИХ КОЛЕБАНИЙ ТВЕРДОГО ТЕЛА, ПОРОЖДАЕМАЯ НЕЛИНЕЙНЫМИ РЕЗОНАНСАМИ
    § 1. Природа неустойчивости вынужденных колебаний твердого тела в условиях нелинейных резонансов
    § 2. Постановка задачи и метод исследования пространственной неустойчивости колебаний твердого тела
    § 3. Пространственная устойчивость колебаний твердого тела в нерезонансном случае
    § 4. Пространственная устойчивость колебаний твердого тела в условиях резонансов
    § 5. Пространственная устойчивость колебаний твердого тела в условиях резонансов при кратных собственных частотах
    § 6. Обобщение критериев устойчивости
    § 7. О взаимной зависимости угловых и поступательных колебаний твердого тела
    § 8. Пространственная устойчивость колебаний твердого тела в условиях комбинационных резонансов
    § 9. Пространственная устойчивость колебаний твердого тела при почти периодических внешних силах
    ГЛАВА V. НЕКОТОРЫЕ ПРИКЛАДНЫЕ ЗАДАЧИ ПРОСТРАНСТВЕННОЙ УСТОЙЧИВОСТИ КОЛЕБАНИЙ ТВЕРДОГО ТЕЛА
    § 1. Пространственная устойчивость колебаний твердого тела, установленного на вибрирующем основании
    2. Условия пространственной устойчивости и их анализ.
    § 2. Пространственная устойчивость колебаний амортизированного объекта
    § 3. Анализ конструкции некоторых типов вибрационных машин
    ГЛАВА VI. ПЕРИОДИЧЕСКИЕ РЕЖИМЫ РЕЗОНАНСНЫХ ПРОСТРАНСТВЕННЫХ КОЛЕБАНИЙ ТВЕРДОГО ТЕЛА ОКОЛО ЦЕНТРА МАСС
    § 1. Периодические колебания твердого тела около центра масс. Одночастотные резонансы
    § 2. Периодические режимы колебаний твердого тела около центра масс в случае кратных резонансов
    ГЛАВА VII. ПЕРИОДИЧЕСКИЕ И ПОЧТИ-ПЕРИОДИЧЕСКИЕ РЕЖИМЫ ПРОСТРАНСТВЕННЫХ КОЛЕБАНИЙ СВОБОДНОГО ТВЕРДОГО ТЕЛА
    § 1. Постановка задачи. Периодические режимы пространственных колебаний в области субгармонического резонанса
    § 2. Интегральные кривые и формы пространственных движений тела
    § 3. Периодические режимы пространственных колебаний тела в областях одночастотных резонансов n-го рода (n=1, 1/2, 1/3)
    § 4. Периодические режимы колебаний свободного твердого тела в случае кратных частот. Взаимодействие поступательных и угловых движений тела
    § 5. Устойчивость периодических движений. Трехмерные и четырехмерные движения тела
    § 6. Почти периодические режимы пространственных колебаний твердого тела и устойчивость движения
    ГЛАВА VIII. НЕЛИНЕЙНЫЕ РЕЗОНАНСНЫЕ ЯВЛЕНИЯ В СИСТЕМАХ ТВЕРДЫХ ТЕЛ, ИМЕЮЩИХ ВРАЩАЮЩИЕСЯ И КОЛЕБЛЮЩИЕСЯ ЧАСТИ
    § 1. Уравнения движения. Постановка задачи
    § 2. Анализ линейных уравнений движения. Области устойчивости и неустойчивости
    § 3. Построение приближенного решения и исследование устойчивости
    § 4. Периодические колебания вращающихся твердых тел. Взаимодействие поступательных и угловых движений тела
    § 5. Почти периодические колебания вращающихся тел
    ГЛАВА IX. ЭКСПЕРИМЕНТАЛЬНОЕ ИССЛЕДОВАНИЕ НЕЛИНЕЙНЫХ ПРОСТРАНСТВЕННЫХ КОЛЕБАНИЙ ТВЕРДЫХ ТЕЛ
    § 1. Описание механических моделей
    § 2. Возбудительная и измерительная аппаратура. Методика измерения колебаний
    § 3. Экспериментальное исследование пространственной устойчивости движения твердых тел
    § 4. Экспериментальное исследование нелинейных резонансных режимов пространственных колебаний твердых тел
    ГЛАВА X. НЕЛИНЕЙНЫЕ ПРОСТРАНСТВЕННЫЕ КОЛЕБАНИЯ СПУТНИКОВ. РЕЗОНАНСНОЕ ВЗАИМОДЕЙСТВИЕ ПОСТУПАТЕЛЬНЫХ И ВРАЩАТЕЛЬНЫХ ДВИЖЕНИЙ СПУТНИКА
    § 1. Уравнения движения при нелинейных пространственных колебаниях спутника относительно центра масс. Малые пространственные колебания спутника
    § 2. Нелинейные пространственные колебания спутника относительно центра масс
    § 3. О взаимосвязи поступательного и вращательного колебательного движений твердого тела в ньютоновском поле сил
    § 4. О механической модели, имитирующей колебательное движение твердого тела в ньютоновском центральном поле сил
    ГЛАВА XI. УСТОЙЧИВОСТЬ ДВИЖЕНИЯ ГИРОСКОПИЧЕСКИХ СИСТЕМ В УСЛОВИЯХ РЕЗОНАНСОВ
    § 1. Обобщенная гироскопическая система на подвижном основании
    § 2. Трехстепенной гироскоп в кардановом подвесе
    § 3. Двухосная гирорама на вибрирующем основании
    2. Анализ областей неустойчивости двухосной гирорамы при комбинационных резонансах.
    § 4. Трехосная гирорама
    ГЛАВА XII. ПРОСТРАНСТВЕННАЯ УСТОЙЧИВОСТЬ ДВИЖЕНИЯ УПРАВЛЯЕМЫХ ТЕЛ. НЕЛИНЕЙНЫЕ КОЛЕБАНИЯ ВЕРТОЛЕТА И АВИАЦИОННЫХ ДВИГАТЕЛЕЙ
    § 1. Пространственная устойчивость движения управляемого тела
    § 2. Пространственная устойчивость движения управляемого спутника
    § 3. Пространственная устойчивость и нелинейные колебания вертолета
    § 4. Выбор рациональной подвески авиационного двигателя на самолете
    ПРИЛОЖЕНИЯ
    ПРИЛОЖЕНИЕ 1. ОПРЕДЕЛЕНИЕ СИЛ И МОМЕНТОВ РЕАКЦИИ УПРУГИХ ПРУЖИН
    ПРИЛОЖЕНИЕ 2
    ПРИЛОЖЕНИЕ 4. Дифференциальные уравнения колебательного движения некоторых технических объектов
    ЛИТЕРАТУРА

    Таблица косинусов (C) — LiteratePrograms

    Из LiteratePrograms

    Перейти к: навигация, поиск

    Эта программа представляет собой дамп кода .

    Tg 360 градусов равен: tg 360 градусов

    Mathway | Популярные задачи

    1Найти точное значениеsin(30)
    2Найти точное значениеsin(45)
    3Найти точное значениеsin(30 град. )
    4Найти точное значениеsin(60 град. )
    5Найти точное значениеtan(30 град. )
    6Найти точное значениеarcsin(-1)
    7Найти точное значениеsin(pi/6)
    8Найти точное значениеcos(pi/4)
    9Найти точное значениеsin(45 град. )
    10Найти точное значениеsin(pi/3)
    11Найти точное значениеarctan(-1)
    12Найти точное значениеcos(45 град. )
    13Найти точное значениеcos(30 град. )
    14Найти точное значениеtan(60)
    15Найти точное значениеcsc(45 град. )
    16Найти точное значениеtan(60 град. )
    17Найти точное значениеsec(30 град. )
    18Найти точное значениеcos(60 град. )
    19Найти точное значениеcos(150)
    20Найти точное значениеsin(60)
    21Найти точное значениеcos(pi/2)
    22Найти точное значениеtan(45 град. )
    23Найти точное значениеarctan(- квадратный корень из 3)
    24Найти точное значениеcsc(60 град. )
    25Найти точное значениеsec(45 град. )
    26Найти точное значениеcsc(30 град. )
    27Найти точное значениеsin(0)
    28Найти точное значениеsin(120)
    29Найти точное значениеcos(90)
    30Преобразовать из радианов в градусыpi/3
    31Найти точное значениеtan(30)
    32Преобразовать из градусов в радианы45
    33Найти точное значениеcos(45)
    34Упроститьsin(theta)^2+cos(theta)^2
    35Преобразовать из радианов в градусыpi/6
    36Найти точное значениеcot(30 град. )
    37Найти точное значениеarccos(-1)
    38Найти точное значениеarctan(0)
    39Найти точное значениеcot(60 град. )
    40Преобразовать из градусов в радианы30
    41Преобразовать из радианов в градусы(2pi)/3
    42Найти точное значениеsin((5pi)/3)
    43Найти точное значениеsin((3pi)/4)
    44Найти точное значениеtan(pi/2)
    45Найти точное значениеsin(300)
    46Найти точное значениеcos(30)
    47Найти точное значениеcos(60)
    48Найти точное значениеcos(0)
    49Найти точное значениеcos(135)
    50Найти точное значениеcos((5pi)/3)
    51Найти точное значениеcos(210)
    52Найти точное значениеsec(60 град. )
    53Найти точное значениеsin(300 град. )
    54Преобразовать из градусов в радианы135
    55Преобразовать из градусов в радианы150
    56Преобразовать из радианов в градусы(5pi)/6
    57Преобразовать из радианов в градусы(5pi)/3
    58Преобразовать из градусов в радианы89 град.
    59Преобразовать из градусов в радианы60
    60Найти точное значениеsin(135 град. )
    61Найти точное значениеsin(150)
    62Найти точное значениеsin(240 град. )
    63Найти точное значениеcot(45 град. )
    64Преобразовать из радианов в градусы(5pi)/4
    65Найти точное значениеsin(225)
    66Найти точное значениеsin(240)
    67Найти точное значениеcos(150 град. )
    68Найти точное значениеtan(45)
    69Вычислитьsin(30 град. )
    70Найти точное значениеsec(0)
    71Найти точное значениеcos((5pi)/6)
    72Найти точное значениеcsc(30)
    73Найти точное значениеarcsin(( квадратный корень из 2)/2)
    74Найти точное значениеtan((5pi)/3)
    75Найти точное значениеtan(0)
    76Вычислитьsin(60 град. )
    77Найти точное значениеarctan(-( квадратный корень из 3)/3)
    78Преобразовать из радианов в градусы(3pi)/4
    79Найти точное значениеsin((7pi)/4)
    80Найти точное значениеarcsin(-1/2)
    81Найти точное значениеsin((4pi)/3)
    82Найти точное значениеcsc(45)
    83Упроститьarctan( квадратный корень из 3)
    84Найти точное значениеsin(135)
    85Найти точное значениеsin(105)
    86Найти точное значениеsin(150 град. )
    87Найти точное значениеsin((2pi)/3)
    88Найти точное значениеtan((2pi)/3)
    89Преобразовать из радианов в градусыpi/4
    90Найти точное значениеsin(pi/2)
    91Найти точное значениеsec(45)
    92Найти точное значениеcos((5pi)/4)
    93Найти точное значениеcos((7pi)/6)
    94Найти точное значениеarcsin(0)
    95Найти точное значениеsin(120 град. )
    96Найти точное значениеtan((7pi)/6)
    97Найти точное значениеcos(270)
    98Найти точное значениеsin((7pi)/6)
    99Найти точное значениеarcsin(-( квадратный корень из 2)/2)
    100Преобразовать из градусов в радианы88 град.

    Дальность HIMARS почти удвоится. Рассказываем о бомбах GLSDB, для которых не нужен самолет и которые США, вероятно, поставят Украине

    По информации СМИ, в новый пакет помощи Украине от США войдет крайне необычное оружие — планирующие авиабомбы GLSDB, которые запускаются не с самолетов, а с помощью реактивных систем залпового огня вроде HIMARS. Рассказываем, как получилось создать такое оружие и чего от него можно ждать. Если кратко, дальнобойность украинского оружия еще больше вырастет — фактически в два раза.

    Что произошло

    Соединенные Штаты Америки готовят новую военную помощь Украине на сумму более двух миллиардов долларов. Об этом пишет информагентство Reuters, ссылаясь на слова двух неназванных американских чиновников.

    Об очередном (уже 32-м по счету с августа 2021 года) пакете военной поддержки будет официально объявлено уже на этой неделе, сообщили источники Reuters. По их информации, в списке поставок ожидается оборудование для зенитно-ракетных комплексов большой дальности Patriot, противотанковые «Джавелины» и высокоточные боеприпасы.

    Один из чиновников указал, что часть средств на военную поддержку украинской армии, а именно 1,725 миллиарда долларов, будет выделена из фонда «Инициатива содействия безопасности Украины» (Ukraine Security Assistance Initiative, USAI), это позволит администрации Джо Байдена получать вооружения не из запасов армии США, а напрямую закупать их у предприятий военной промышленности.

    Главным разочарованием оказался отказ США удовлетворить запрос Украины послать ВСУ дальнобойные ракеты ATACMS для РСЗО HIMARS c дальностью почти в 300 километров, которыми можно было бы обстреливать военные объекты в глубоком тылу российских войск. Главным же плюсом пакета помощи, как ни странно, оказалось также дальнобойное оружие, запускаемое из HIMARS — а именно гибрид высокоточных планирующих бомб GBU-39 и ракетных двигателей от неуправляемых снарядов M26, которых на складах в США достаточно. Звучит сложно, давайте разбираться.

    Что такое бомбы GBU-39

    В начале XXI века авиационные гиганты из США Boeing и Lockheed Martin получили контракт на создание высокоточной бомбы малого диаметра (SDB — Small Diameter Bomb). Это можно назвать американским трендом развития вооружений — конструкторы в США создают оружие со сравнительно скромной разрушающей способностью, однако попадающее точно в цель.

    Такой подход позволяет, с одной стороны, избежать лишних жертв при ударах (к примеру, в городской застройке), с другой — эффективно оказывать войскам поддержку высокоточными выстрелами (бомбовый или ракетный удар можно вызвать по врагам, находящимся в считанных десятках метров от собственных солдат, и при этом дружественные войска не пострадают, так как ни взрывная волна, ни осколки их не поразят).

    Корректируемая бомба GBU-39 планирует на цель. Изображение: saab.com

    На разработку ушло пять лет, осенью 2006 года были готовы первые экземпляры, получившие название GBU-39. Практически тут же «Боинг» создал вариант, еще более снижавший ущерб посторонним (он назывался FLM — Focused Lethality Munition, «боеприпас сфокусированной смертоносности»). Для того, чтобы бомба уничтожала только тех, на кого направлена, стальной корпус (сам по себе представляющий собой опасность — разлетающийся от взрыва металл может ранить окружающих) заменили на углепластиковый, а боеприпас адаптировали для направленного взрыва — но этот вариант пока экспериментальный.

    Уже в октябре 2006 года бомбу впервые применили во время кампании в Ираке. Боеприпас использовали с ударной модификации тяжелого истребителя F-15 — F-15E Strike Eagle. Небольшая бомба имела массу чуть более 120 кг, крылья, позволяющие планировать на расстояние до 110 километров (если сбросить ее с большой высоты порядка 10 километров) и классическую для нашего времени инерциальную навигацию со спутниковой коррекцией (этот термин означает, что в район цели бомба летит по автопилоту, а на финальном участке траектории подруливает, чтобы точно попасть в объект, ориентируясь по сигналам спутника). Даже самые первые бомбы имели круговое вероятное отклонение или КВО около пяти метров (то есть половина сброшенных GBU-39 укладывается в круг пятиметрового радиуса), более совершенные версии имели КВО в один метр.

    А при чем тут HIMARS, если бомба сбрасывается с самолета?

    Прошлой весной, когда появились первые слухи о поставках в Украину реактивных систем залпового огня HIMARS, не было понятно, какие именно ракеты поставят в комплекте с пусковыми установками и будет ли их огонь точным. «Хаймарсы» не всегда имели корректируемые боеприпасы — к примеру, снаряды M26 c кассетной боевой частью были неуправляемыми.

    Тогда мы предположили, что M26 Вашингтон поставлять не будет, так как иракская кампания показала их низкую эффективность и опасность для дружественных войск, а вместо этого отправит Киеву сверхточные M31. Так и вышло — Украина получила «снайперскую винтовку» дальностью в 80 километров, которая и изменила ход войны. А M26 украинцам не отправили. Но отправят, пусть и не целиком.

    Дело в том, что признанные негодными неуправляемые снаряды по-прежнему располагают вполне рабочими реактивными двигателями, позволяющими доставлять смертоносный груз на десятки километров. Эти силовые установки уже произведены и лежат на складах мертвым грузом. Оружейники задумались: а почему бы не использовать в качестве боевой части бомбу?

    Идея очень простая. На большую высоту бомбу GBU-39, разработанную для самолетов, совершенно необязательно поднимать с помощью F-15E или другого бомбардировщика или истребителя — это может сделать ракетный двигатель M26, поработав в роли этакого одноразового самолета. Набрав высоту и скорость и отстыковавшись от двигателя, бомба может использоваться по назначению.

    В разработке «ракетобомбы» участвовали Boeing и шведский оборонный гигант Saab. В конце зимы 2015 прошли три испытания нового-старого боеприпаса, все оказались успешными. Получившееся оружие назвали GLSDB (Ground Launched Small Diameter Bomb — «бомба малого диаметра, запускаемая с земли»)

    Оказалось, что запущенная из «Хаймарса» бомба может даже разворачиваться на 360 градусов и атаковать цели, которые остались позади. Дальность полета боеприпаса оказалась огромной: сначала 100 километров, потом 130 и наконец 150. При этом он может в случае перелета вернуться назад на огромное расстояние в 70 километров благодаря исключительной способности разворачиваться и планировать.

    Впервые о возможности поставить Украине GLSDB заговорили в ноябре прошлого года, когда производить такие боеприпасы для Киева предложили представители «Боинга», а Пентагон согласился рассмотреть идею. Тогда особо отмечалась дешевизна нового оружия: авиабомба GBU-39 стоит 40 тысяч долларов, цена ракетных двигателей значения не имеет, так как они все равно уже произведены — таким образом, конечная стоимость боеприпаса будет равна 40 000 плюс стоимость установки бомбы на двигатель и транспортировки (это удивительно дешево для дальнобойной «высокоточки» — тот же ATACMS стоит под миллион долларов).

    Что даст Украине новая бомба?

    Долгожданную «длинную руку». Сейчас никаких способов наносить регулярные удары за пределами восьмидесятикилометровой зоны поражения «Хаймарсов» ВСУ не демонстрирует — поставленные ранее противорадарные ракеты являются редким нишевым оружием, крылатые ракеты из беспилотников «Стриж» дальнобойны, но редки, а комплекты JDAM пока никак себя, насколько можно судить, не проявили.

    Получив недорогой боеприпас для запуска из давно освоенной системы залпового огня, ВСУ смогут с высокой точностью уничтожать разведанные объекты в значительно более глубоком тылу российских войск. Исследователь по открытым источникам Def Mon выложил в своем твиттере сравнительную зону поражения обычных ракет «Хаймарса» и планирующих бомб. Последний оказался намного больше и включает все территории, оккупированные Россией в материковой части Украины (включая сухопутный коридор в Крым), северную часть Крымского полуострова (оккупированного с 2014 года) и часть Ростовской области с городом Таганрогом.

    Visual of the approximate area which will get in range if Ukraine are provided with 150 km standoff weapons.

    This might be happening according to Reuters. pic.twitter.com/hW4iHpI3zh

    — Def Mon (@DefMon3) January 31, 2023

    В случае, если украинские войска пойдут в наступление, удары GLSDB могут предварить удар, проведя изоляцию района боевых действий — то есть уничтожив вражеские силы и военный потенциал, который может помешать наступать (это называется interdiction mission — одна из наиболее эффективных тактик армий НАТО, похожая на ту, для которой создавались ракеты ATACMS). А предполагаемая дешевизна бомб, вероятно, позволит поставлять их более-менее массово (что, впрочем, необязательно, так как производство таких боеприпасов, как бы просто оно ни было, только разворачивается).

    Не все вооружения, поставленные Украине Западом, работают так, как предполагалось. Однако, судя по заявленным характеристикам GLSDB, шансы проявить себя у этой модели есть.

    Читайте также

    Tan 360 градусов — Найдите значение Tan 360 градусов

    LearnPracticeDownload

    Значение tan 360 градусов равно 0 . Тангенс 360 градусов в радианах записывается как тангенс (360° × π/180°), то есть тангенс (2π) или тангенс (6,283185…). В этой статье мы обсудим методы определения значения tan 360 градусов на примерах.

    • Желто-коричневый 360°: 0
    • Желто-коричневый (-360 градусов): 0
    • Tan 360° в радианах: тан (2π) или тан (6,2831853 . . . .)

    Сколько стоит Тан 360 градусов?

    Значение тангенса 360 градусов равно 0. Тангенс 360 градусов также может быть выражен с помощью эквивалента данного угла (360 градусов) в радианах (6,28318 . . .)

    Мы знаем, используя преобразование градусов в радианы, θ в радианы = θ в градусах × (pi/180°)
    ⇒ 360 градусов = 360° × (π/180°) рад = 2π или 6,2831. . .
    ∴ tan 360° = tan(6.2831) = 0

    Объяснение:

    Для tan 360 градусов угол 360° лежит на положительной оси x. Таким образом, значение tan 360° = 0
    Поскольку функция тангенса является периодической функцией, мы можем представить тангенс 360° как тангенс 360 градусов = тангенс (360° + n × 180°), n ∈ Z.
    ⇒ тангенс 360° = тангенс 540° = тангенс 720° и так далее.
    Примечание: Поскольку тангенс является нечетной функцией, значение тангенса (-360°) = -тангенса (360°) = 0.

    Методы определения значения тангенса 360 градусов

    Значение тангенса 360° задается как 0. Мы можем найти значение тангенса 360 градусов по:

    • Использование единичного круга
    • Использование тригонометрических функций

    Tan 360 градусов с использованием единичного круга

    Чтобы найти значение tan 360 градусов с помощью единичного круга:

    • Поверните ‘r’ против часовой стрелки, чтобы образовать угол 0° или 360° с положительной осью x.
    • Тангенс 360 градусов равен координате y(0), деленной на координату x(1) точки пересечения (1, 0) единичной окружности и r.

    Следовательно, значение tan 360° = y/x = 0

    Тангенс 360° в терминах тригонометрических функций

    Используя формулы тригонометрии, мы можем представить тангенс 360° как:

    • sin(360°)/cos(360°)
    • ± sin 360°/√(1 — sin²(360°))
    • ± √(1 — cos²(360°))/cos 360°
    • ± 1/√(косек²(360°) — 1)
    • ± √(сек²(360°) — 1)
    • 1/кроватка 360°

    Мы можем использовать тригонометрические тождества для представления tan 360° как

    • cot(90° — 360°) = раскладушка(-270°)
    • -кроватка(90° + 360°) = -кроватка 450°
    • -tan (180° — 360°) = -tan(-180°)

    Примечание. Поскольку 360° лежит на положительной оси x, окончательное значение тангенса 360° равно 0.

    ☛ Также проверьте:

    • тангенс 105 градусов
    • загар 270 градусов
    • загар 135 градусов
    • загар 900 градусов
    • загар 21 градус
    • тан 11 градусов

    Примеры использования Tan 360 градусов

    1. Пример 1. Найдите значение 4 тангенса (360°)/6 тангенса (45°).

      Решение:

      Используя тригонометрические значения, мы знаем, что tan(360°) = 0 и tan 45° = 1.
      ⇒ Значение тангенса 4 (360°)/тангенса 6 (45°) = 0

    2. Пример 2: Упростить: 6 (tan 360°/cot(225°))

      Решение:

      Мы знаем tan 360° = 0 и cot 225° = 1
      ⇒ 6 тан 360°/кот 225° = 6(0)
      = 0

    3. Пример 3. Найдите значение (2 sin (180°) cos (180°) sec (360°)). [Подсказка: используйте tan 360° = 0]

      Решение:

      Используя формулу sin 2a,
      2 sin (180°) cos (180°) = sin (2 × 180°) = sin 360°
      ⇒ 2 sin (180°) cos (180°) сек(360°) = sin 360° сек 360°
      = sin 360°/cos 360° = tan 360°
      ⇒ (2 sin (180°) cos (180°) сек(360°)) = 0

    перейти к слайдуперейти к слайдуперейти к слайду

     

    Готовы увидеть мир глазами математика?

    Математика лежит в основе всего, что мы делаем. Наслаждайтесь решением реальных математических задач на живых уроках и станьте экспертом во всем.

    Запишитесь на бесплатный пробный урок

    Часто задаваемые вопросы о Tan 360 Degrees

    Что такое Tan 360 Degrees?

    Tan 360 градусов — значение тригонометрической функции тангенса для угла, равного 360 градусам. Значение tan 360° равно 0.

    Какое значение Tan 360° в терминах Sec 360°?

    Мы можем представить функцию тангенса в терминах функции секущей, используя тригонометрические тождества, тангенс 360° можно записать как √(sec²(360°) — 1). Здесь значение sec 360° равно 1.

    Каково значение Tan 360 градусов в терминах Cot 360°?

    Поскольку функция тангенса является обратной функцией котангенса, мы можем записать тангенс 360° как 1/cot(360°).

    Как найти значение Tan 360 градусов?

    Значение tan 360 градусов можно рассчитать, построив угол 360° с осью x и затем найдя координаты соответствующей точки (1, 0) на единичной окружности. Значение tan 360° равно координате y (0), деленной на координату x (1). ∴ tan 360° = 0

    Как найти Tan 360° с точки зрения других тригонометрических функций?

    Используя формулу тригонометрии, значение тангенса 360° может быть выражено через другие тригонометрические функции следующим образом:

    • sin(360°)/cos(360°)
    • ± sin 360°/√(1 — sin²(360°))
    • ± √(1 — cos²(360°))/cos 360°
    • ± 1/√(косек²(360°) — 1)
    • ± √(сек²(360°) — 1)
    • 1/кроватка 360°

    ☛ Также проверьте: таблицу тригонометрии

     

    Скачать БЕСПЛАТНО учебные материалы

    Тригонометрия

    Рабочие листы по математике и
    наглядный учебный план

    Тригонометрия В играх Так что

    вы хотите сделать некоторые крутые эффекты в игре; какая-то съемка на 360 градусов, может быть синусоидальные волны и, может быть, вращающиеся объекты. При обычных 32-направленных движениях MMF2 это невозможно. Вы можете стать точным, но вы не сможете приблизиться к красивым, профессиональным движениям, которым можно легко научиться.

    Введение


    Я не прошу много времени, это очень простой урок, и я все объясню. Пожалуйста, если вы планируете изучать 360-градусную стрельбу или тригонометрию, продолжайте читать. Вы не найдете более короткого метода, так что просто потратьте 15 минут и готово. Чтобы учиться, у вас должно быть свободное время.

    В этом уроке мы рассмотрим синус, косинус, тангенс и арктангенс. Нам не нужно будет использовать какие-либо другие тригонометрические функции.

    Для этого урока необходимо базовое понимание MMF2. Если вам трудно понять действия и то, как мы управляем нашими объектами, возможно, этот урок вам не подходит. Это несколько продвинутое руководство для тех, кто хочет иметь больше мощности и контроля в своих играх.
    Теперь, если у вас есть проблемы с пониманием математики и выражений, не бросайте, а попытайтесь понять. Найди меня, Элиягу, и я помогу тебе.

    Теперь, когда вступление сделано…. К ОБУЧЕНИЮ!

    Треугольник


    Рассмотрим прямоугольный треугольник ABC. Его углы равны A, B и C, а его стороны равны x, y и z. Обычно стороны обозначаются строчной буквой противоположного угла, но для простоты понимания мы будем использовать x, y и z:

    Теперь немного по-другому рассмотрим текст SOHCAHTOA. Это маленькое «слово» всегда будет важным, так как оно описывает отношения между углами и двумя сторонами.

    • S=O/H C=A/H T=O/A 92, Теорему Пифагора, следует повторить вам, поэтому мы ее пропустим. Противоположная, как сказано, сторона, противоположная углу. Противоположностью C является z. Прилегающая сторона — это сторона, которая находится рядом или касается угла, но НЕ гипотенуза. Сторона, примыкающая к С, равна х.

      Принимая во внимание вышеизложенное, мы можем многое узнать об угле. Например: (если вы не поняли, Cos=косинус, Sin=синус, Tan=тангенс)

      • Cos(A) = z/y
      • Кос(С) = х/у
      • Sin(A) = x/y
      • Sin(C) = z/y
      • Тан(А) = x/z
      • Тан(С) = z/x

      И даже не пытайся использовать тригонометрические функции для прямого угла, B, это не работает. Это может быть много, чтобы взять все сразу, но на самом деле это очень просто. У вас есть свой набор правил, SOHCAHTOA, поэтому, если вы хотите найти синус, косинус или тангенс угла, вы можете это сделать!

      ПРИМЕЧАНИЕ. Вам не нужно запоминать функции, относящиеся к нашему треугольнику, потому что с SOHCATOA вы можете найти их в любое время. Ключ SOHCATOA, и вы можете найти его для ЛЮБОГО прямоугольного треугольника.

      Atan2


      Теперь, когда у нас есть общее представление о трех основных тригонометрических функциях, давайте взглянем на арксинус, арккосинус и арктангенс. Допустим, у нас есть 2 стороны треугольника, теперь нам нужен угол. Раньше мы брали угол, выполняли над ним функцию и получали из него число.

      • Рассмотрим: Cos(A)=z/y

      Зная z и y, мы можем сказать, что угол A можно найти, когда косинус равен z/y. Делаем это с помощью арккосинуса. Обозначается как ACos (также ASin и ATan). Например, вы бы решили это как:

      • А = ACos(z/y)

      Это говорит о том, что пусть A равно значению, когда косинус этого значения равен z/y. Для игр нам нужно беспокоиться только об ATan прямо сейчас, но мы доберемся до этого. Чтобы держать это в уме, помните следующее, относящееся к треугольнику выше:

      • C = ATan(z/x)

      Это скоро вступит в игру…  А теперь давайте сосредоточимся на реализации этого в игре.

      Треугольник мыши


      Стрельба на 360 градусов — наиболее распространенный триггер в играх. Итак, представьте позиции X и Y объекта и вашей мыши. Мы хотим выстрелить в объект в этом направлении. Понятно, что x = Xmouse-X («Объект») и y = Ymouse-Y («Объект») Вместе они образуют треугольник, где z является гипотенузой. Если это происходит слишком быстро и вы запутались, не заходите дальше, это только еще больше запутает. Прошу прощения, я делаю все, что могу.

      Итак, мы хотим выпустить снаряд из точки A в направлении C. Очевидно, что нам нужен угол A, чтобы лететь дальше. Это можно было бы сделать с помощью C, но A — более разумный и ясный выбор, поэтому давайте возьмем угол A. Но как? Все, что у нас есть, это x и y, которые являются различиями в позициях x и y нашей мыши по сравнению с объектом. НО мы можем использовать ATan для получения угла A.

      Подумайте об этом, тангенс угла A равен y/x. Следовательно, A = ATan(y/x). Это кажется прекрасным и модным, и так и должно быть, но MMF2 и другие языки позволят нам использовать ATan2, который является единственным вариантом для съемки 360, так что давайте посмотрим на ATan2.

      В MMF2 у нас есть доступ ко всем обычным функциям, ACos, ASin, ATan и ATan2. Элиягу, что это за ATan2?!?! Ну, ATan возвращает только угол в диапазоне от -90 до 90. Это движение на 180 градусов. Этого будет недостаточно для полноценного вращательного шутера. Поэтому мы используем ATan2, который имеет диапазон от -180 до 180, полные 360 градусов круга!

      Забудь об ATan, он тебе действительно не понадобится. Если вы когда-либо не можете решить, какой из них, используйте ATan2. Итак, мы определили, что угол A фактически равен ATan2(y/x). Проблема в том, что MMF2 принимает параметр как параметр y и параметр x. Это означает, что вам нужно ввести 2 отдельных значения: числитель дроби и знаменатель. Это делается для того, чтобы он мог определить угол, поскольку тангенс не может указывать 360 градусов, поэтому MMF2 использует логику положительных и отрицательных значений в направлении, чтобы определить, какой угол будет. Если вы изучите единичный круг, вы поймете, но на данный момент вам это не нужно, и он не включен в этот урок. Подводя итог, угол A = ATan2(YMouse-Y(«Объект»), XMouse-X(«Объект»)) Обычно он равен y/x, но, как мы сказали, нам нужны числитель и знаменатель как 2 отдельных параметра.

      Мы знаем угол А. Но что нам в этом хорошего, если мы не можем двигаться под этим углом? Так что давайте прикроем движение в этом направлении. Что мы действительно ищем, так это x и y, по одному, чтобы выяснить, насколько они будут двигаться по этой оси, а также по осям x и y. Считайте z длиной движения. Это гипотенуза и, следовательно, общее количество пройденных пикселей. Если мы забудем указать это в нашем выражении, предполагается, что оно равно 1, и тогда x и y будут меньше или равны 1, и наш объект никуда не денется.

      Учитывая A и z, давайте найдем x. Мы знаем, что Cos(A) = x/z. Умножаем обе части на z, и получаем x = z*Cos(A). Это так просто. Мы можем сделать то же самое для y. Sin(A) = y/z, поэтому y = z*Sin(A) Давайте будем проще и допустим z=10. В этом уроке при создании пули или снаряда мы сохраняем угол в изменяемом значении «гниль». (вращение) Мы добавим к позиции X 10*Cos(rot(«Пуля»)), а к позиции Y 10*Sin(rot(«Пуля»)) Давайте посмотрим, как это будет выглядеть! Другими словами, мы наконец-то увидим экшн!

      360-градусная съемка


      Красный объект, База, несколько центрирован. Он создает объекты, пули. Посмотрите на код, чтобы увидеть, как это работает. (СМ. СКАЧАТЬ НИЖЕ) Нажмите, чтобы стрелять. Далее мы сделаем его немного более динамичным. Мы также немного очистим его, потому что вы заметите, что все не отцентровано, и пуля оказывается немного не там, где вы хотите.

      На этот раз Пуля имеет изменяемое значение ‘vel’ (скорость), с которым она будет двигаться. Мы можем изменить это, потому что это переменная. Мы больше не ограничены 10!
      Если удерживать левую кнопку мыши, все существующие пули замедлятся. Мы могли бы сделать так, чтобы они инициализировались с такой скоростью из глобального значения, на которое они все обращают внимание при создании, но давайте не будем усложнять. Удерживание правой кнопки добавит вращению, поэтому они будут кривыми.

      ДЕМО-КАДР, ДОСТУПНЫЙ В MFA

      Орбита как спутник с тригонометрией


      Давайте рассмотрим, что мы можем сделать с помощью тригонометрии на данный момент: Мы можем определить угол по разнице x и y. Мы можем определить движение по осям x и y, зная угол и скорость.

      Посмотрим на вторую. До сих пор мы использовали угол A для угла, но что, если мы использовали несвязанный угол? Угол, который всегда увеличивается. Мы можем создать спутник, объект, который вращается вокруг другого.

      Центр нашего спутника (x,y), который также может быть (X(«Объект»), Y(«Объект»)) или (XMouse, YMouse). Если наш спутник имеет изменяемое значение rot, которое мы постоянно добавляем, мы можем установить его положение по оси X как X+75*Cos(rot(«спутник»)), а Y как Y+75*Sin(rot(«спутник»). ), то у нас будет объект на орбите с радиусом 75 пикселей от центра (X, Y).

      Давайте посмотрим, что из этого получится, ниже. Комментарии объясняют, что происходит. В следующем кадре будут небольшие изменения в этом примере.

      ДЕМО-КАДР, ДОСТУПНЫЙ В MFA

      Синус и косинус


      Давайте немного вернемся назад и посмотрим на Sin(x). Вам пока не нужно понимать единичный круг, если только вы не хотите очень специфических волн. Пока вы можете понять следующее… вам даже не нужно знать это. Просто знайте, как реагирует синусоида:

       

      • Sin(0)=0
      • Sin(45)=sqrt(2)/2
      • Sin(90)=1
      • Sin(135)=sqrt(2)/2
      • Sin(180)=0,
      • Sin(225)=-sqrt(2)/2
      • Sin(270)=-1
      • Sin(315)=-sqrt(2)/2
      • Sin(360)=0

      Это будет повторяться даже после того, как вы превысите 360.

    Объем сосуда: Расчет объема сосуда — онлайн калькулятор

    нюансы расчета объема жидкости в зависимости от формы емкости

    Содержание:

    • Что такое вместимость сосуда
    • Особенности расчета объема жидкости в сосуде
    • Как определить вместимость сосудов разных форм
      • Вычисление объема параллелепипеда
      • Нахождение объема пирамиды
      • Как найти объем цилиндра
      • Как высчитать объем конуса
      • Нахождение объема шара

    Содержание

    • Что такое вместимость сосуда
    • Особенности расчета объема жидкости в сосуде
    • Как определить вместимость сосудов разных форм
      • Вычисление объема параллелепипеда
      • Нахождение объема пирамиды
      • Как найти объем цилиндра
      • Как высчитать объем конуса
      • Нахождение объема шара

    Что такое вместимость сосуда

    Вместимость сосуда — это объем его внутренней полости, определяемый по его геометрическим параметрам. Единица измерения объема в СИ — кубический метр, но в случае жидкости чаще используют литр.

    Особенности расчета объема жидкости в сосуде

    Жидкость по своим свойствам занимает промежуточное место между двумя другими агрегатными состояниями вещества — твердым и газообразным. Жидкости присущи некоторые свойства и твердого тела, и газа. Силы взаимного притяжения молекул в жидкостях достаточно велики, чтобы удерживать молекулы вместе, так что, в отличие от газов, жидкости имеют постоянный собственный объем.

    В то же время эти силы недостаточны, чтобы держать молекулы в жесткой упорядоченной структуре, и потому у жидкостей нет постоянной формы: они принимают форму сосуда, в котором находятся.

    Осторожно! Если преподаватель обнаружит плагиат в работе, не избежать крупных проблем (вплоть до отчисления). Если нет возможности написать самому, закажите тут.

    Жидкость в сосуде оказывает постоянное давление на его стенки, поэтому на производстве, где необходимо регулярно измерять текущий объем жидкости в сосуде, часто используют гидростатические датчики давления.

    За счет маленького диаметра их мембран итоговая погрешность измерения близится к нулю. Поэтому, зная давление в конкретный момент времени, можно вычислять уровень жидкости, т. е. высоту гидростатического столба. В формулу для расчета входят только плотность жидкости и ее давление:

    \(h = \frac{p}{\rho \times g}.\)

    \(p\) здесь — давление в паскалях, \(\rho\) — плотность, \(g\) — ускорение свободного падения, константа.

    Зная габариты сосуда, несложно рассчитать объем жидкости в нем. Это необходимо, например, в пивоварении и виноделии, где обычно используются цилиндрические емкости с конусным дном, близкие по параметрам к идеальным геометрическим телам.

    При решении логических учебных задач на переливание жидкости из одного сосуда в другой может пригодиться понимание взаимосвязи объема жидкости и параметров сосуда. А для задач по физике часто требуется рассчитать объем, который занимает жидкость в сосуде, через ее массу. На практике это действительно один из самых удобных способов, не требующий ни специальных датчиков, ни сложных расчетов. {3}}{3}.\)

    Насколько полезной была для вас статья?

    Рейтинг: 5.00 (Голосов: 1)

    Выделите текст и нажмите одновременно клавиши «Ctrl» и «Enter»

    Поиск по содержимому

    Объем Сосуда, Выраженный В Литрах 6 Букв

    Решение этого кроссворда состоит из 6 букв длиной и начинается с буквы Л


    Ниже вы найдете правильный ответ на Объем сосуда, выраженный в литрах 6 букв, если вам нужна дополнительная помощь в завершении кроссворда, продолжайте навигацию и воспользуйтесь нашей функцией поиска.

    ответ на кроссворд и сканворд

    Суббота, 27 Апреля 2019 Г.



    ЛИТРАЖ

    предыдущий следующий



    ты знаешь ответ ?

    ответ:

    связанные кроссворды

    1. Литраж
      1. Кубатура двигателя
      2. Рабочий объем всех цилиндров двигателя внутреннего сгорания
    2. Литраж
      1. Вместимость чего-нибудь в литрах 6 букв
      2. Метраж, тоннаж, а на троих всегда 6 букв
      3. Рабочий объем всех цилиндров двигателя автомобиля 6 букв
      4. Рабочий объем всех цилиндров двигателя внутреннего сгорания 6 букв

    похожие кроссворды

    1. Объем чего-нибудь, выраженный в кубических единицах 8 букв
    2. Объем выраженный в кубических метрах 8 букв
    3. Внутренний объем или вместимость сосуда
    4. Объём бака или объём двигателя 6 букв
    5. Выраженный дебил
    6. Резко выраженный индивидуализм
    7. Безмолвный, не выраженный словами
    8. Ярко выраженный со стороны какого-нибудь отрицательного качества 8 букв
    9. Сильно, ясно выраженный; напряженный 6 букв
    10. Неясный, неопределенно выраженный 12 букв
    11. Слабо выраженный, неотчетливый, неясный 7 букв
    12. Результат чего-нибудь (например игры), выраженный в числах 4 буквы
    13. Показатель, расчет чего-нибудь, выраженный в числах 5 букв
    14. Выраженный в каком-нибудь количестве, количественный 9 букв

    404 — СТРАНИЦА НЕ НАЙДЕНА

    Почему я вижу эту страницу?

    404 означает, что файл не найден. Если вы уже загрузили файл, имя может быть написано с ошибкой или файл находится в другой папке.

    Другие возможные причины

    Вы можете получить ошибку 404 для изображений, поскольку у вас включена защита от горячих ссылок, а домен отсутствует в списке авторизованных доменов.

    Если вы перейдете по временному URL-адресу (http://ip/~username/) и получите эту ошибку, возможно, проблема связана с набором правил, хранящимся в файле .htaccess. Вы можете попробовать переименовать этот файл в .htaccess-backup и обновить сайт, чтобы посмотреть, решит ли это проблему.

    Также возможно, что вы непреднамеренно удалили корневую папку документа или ваша учетная запись должна быть создана заново. В любом случае, пожалуйста, немедленно свяжитесь с вашим веб-хостингом.

    Вы используете WordPress? См. Раздел об ошибках 404 после перехода по ссылке в WordPress.

    Как найти правильное написание и папку

    Отсутствующие или поврежденные файлы

    Когда вы получаете ошибку 404, обязательно проверьте URL-адрес, который вы пытаетесь использовать в своем браузере. Это сообщает серверу, какой ресурс он должен использовать попытка запроса.

    http://example.com/example/Example/help.html

    В этом примере файл должен находиться в папке public_html/example/Example/

    Обратите внимание, что CaSe важен в этом примере. На платформах с учетом регистра e xample и E xample не совпадают.

    Для дополнительных доменов файл должен находиться в папке public_html/addondomain.com/example/Example/, а имена чувствительны к регистру.

    Разбитое изображение

    Если на вашем сайте отсутствует изображение, вы можете увидеть на своей странице поле с красным цветом X , где изображение отсутствует. Щелкните правой кнопкой мыши X и выберите «Свойства». Свойства сообщат вам путь и имя файла, который не может быть найден.

    Это зависит от браузера. Если вы не видите на своей странице поле с красным X , попробуйте щелкнуть правой кнопкой мыши на странице, затем выберите «Просмотреть информацию о странице» и перейдите на вкладку «Мультимедиа».

    http://example.com/cgi-sys/images/banner.PNG

    В этом примере файл изображения должен находиться в папке public_html/cgi-sys/images/

    Обратите внимание, что в этом примере важен CaSe . На платформах с учетом регистра символов PNG и png не совпадают.

    404 Ошибки после перехода по ссылкам WordPress

    При работе с WordPress часто могут возникать ошибки 404 Page Not Found, когда была активирована новая тема или когда были изменены правила перезаписи в файле .htaccess.

    Когда вы сталкиваетесь с ошибкой 404 в WordPress, у вас есть два варианта ее исправления.

    Вариант 1: Исправьте постоянные ссылки
    1. Войдите в WordPress.
    2. В меню навигации слева в WordPress нажмите  Настройки > Постоянные ссылки (Обратите внимание на текущую настройку. Если вы используете пользовательскую структуру, скопируйте или сохраните ее где-нибудь. )
    3. Выберите  По умолчанию .
    4. Нажмите  Сохранить настройки .
    5. Верните настройки к предыдущей конфигурации (до того, как вы выбрали «По умолчанию»). Верните пользовательскую структуру, если она у вас была.
    6. Нажмите  Сохранить настройки .

    Во многих случаях это сбросит постоянные ссылки и устранит проблему. Если это не сработает, вам может потребоваться отредактировать файл .htaccess напрямую.

    Вариант 2. Измените файл .htaccess

    Добавьте следующий фрагмент кода 9index.php$ — [L]
    RewriteCond %{REQUEST_FILENAME} !-f
    RewriteCond %{REQUEST_FILENAME} !-d
    RewriteRule . /index.php [L]

    # Конец WordPress

    Если ваш блог показывает неправильное доменное имя в ссылках, перенаправляет на другой сайт или отсутствуют изображения и стиль, все это обычно связано с одной и той же проблемой: в вашем блоге WordPress настроено неправильное доменное имя.

    Как изменить файл .htaccess

    Файл .htaccess содержит директивы (инструкции), которые сообщают серверу, как вести себя в определенных сценариях, и напрямую влияют на работу вашего веб-сайта.

    Перенаправление и перезапись URL-адресов — это две очень распространенные директивы, которые можно найти в файле .htaccess, и многие скрипты, такие как WordPress, Drupal, Joomla и Magento, добавляют директивы в .htaccess, чтобы эти скрипты могли работать.

    Возможно, вам потребуется отредактировать файл .htaccess в какой-то момент по разным причинам. В этом разделе рассматривается, как редактировать файл в cPanel, но не то, что может потребоваться изменить. статьи и ресурсы для этой информации.)

    Существует множество способов редактирования файла .htaccess
    • Отредактируйте файл на своем компьютере и загрузите его на сервер через FTP
    • Использовать режим редактирования программы FTP
    • Используйте SSH и текстовый редактор
    • Используйте файловый менеджер в cPanel

    Самый простой способ отредактировать файл . htaccess для большинства людей — через диспетчер файлов в cPanel.

    Как редактировать файлы .htaccess в файловом менеджере cPanel

    Прежде чем что-либо делать, рекомендуется сделать резервную копию вашего веб-сайта, чтобы вы могли вернуться к предыдущей версии, если что-то пойдет не так.

    Откройте файловый менеджер
    1. Войдите в cPanel.
    2. В разделе «Файлы» щелкните значок «Диспетчер файлов ».
    3. Установите флажок для  Корень документа для и выберите доменное имя, к которому вы хотите получить доступ, из раскрывающегося меню.
    4. Убедитесь, что установлен флажок Показать скрытые файлы (точечные файлы) «.
    5. Нажмите  Перейти . Файловый менеджер откроется в новой вкладке или окне.
    6. Найдите файл .htaccess в списке файлов. Возможно, вам придется прокрутить, чтобы найти его.
    Для редактирования файла .htaccess
    1. Щелкните правой кнопкой мыши файл . htaccess и выберите  Редактировать код в меню. Кроме того, вы можете щелкнуть значок файла .htaccess, а затем Редактор кода Значок вверху страницы.
    2. Может появиться диалоговое окно с вопросом о кодировании. Просто нажмите Изменить , чтобы продолжить. Редактор откроется в новом окне.
    3. При необходимости отредактируйте файл.
    4. Нажмите  Сохранить изменения в правом верхнем углу, когда закончите. Изменения будут сохранены.
    5. Протестируйте свой веб-сайт, чтобы убедиться, что ваши изменения были успешно сохранены. Если нет, исправьте ошибку или вернитесь к предыдущей версии, пока ваш сайт снова не заработает.
    6. После завершения нажмите Закрыть , чтобы закрыть окно диспетчера файлов.

    Калькулятор преобразования объема

    Базовый калькулятор

    Конвертировать том

    Значение для преобразования:
    действительное число или научная запись

    От: акр-фут (ac ft)баррель (британский) (bl)баррель (нефтяной) (bl)баррель (сухой) (США) (bl)баррель (жидкий) (США) (bl)бушель (британский) (бу)бушель (сухой) (bu)корд (дрова) (шнур)кубический фут (ft³)кубический дюйм (in³)кубический сантиметр (cm³) или (cc)кубический метр (m³)кубическая миля (mi³)кубический ярд (yd³)cup ( завтрак) (c)чашка (канадская) (c)чашка (США) (c)унция (британская жидкость) (oz)унция (американская жидкость) (oz)галлон (имперская) (gal)галлон (США сухой) (gal) галлон (американская жидкость) (gal)gill (имперский) (gi)gill (США) (gi)хогсхед (имперский) (hhd)хогсхед (США) (hhd)литр (л)миллилитр (мл)пек (имперский) (pk )пек (США, сухой) (pk)пинта (англ. ) жидкость) (qt)столовая ложка (Can) (столовая ложка)столовая ложка (Imp) (столовая ложка)столовая ложка (США) (столовая ложка)чайная ложка (Can) (tsp)чайная ложка (Imp) (tsp)чайная ложка (U.S.) (tsp)

    Кому: акр-фут (ac ft)баррель (британский) (bl)баррель (нефтяной) (bl)баррель (сухой) (США) (bl)баррель (жидкий) (США) (bl)бушель (британский) (bu)бушель (сухой) (США) ( bu)корд (дрова) (шнур)кубический фут (ft³)кубический дюйм (in³)кубический см (см³) или (cc)кубический метр (м³)кубическая миля (mi³)кубический ярд (yd³)чашка (завтрак) (c) чашка (канадская) (c)чашка (США) (c)унция (британская жидкость) (oz)унция (американская жидкость) (oz)галлон (британская единица) (галлон)галлон (сухой доллар США) (галлон)галлон (жидкость США) (gal)gill (имперская) (gi)gill (США) (gi)hogshead (имперская) (hhd)hogshead (США) (hhd)литр (л)миллилитр (мл)пек (имперская) (pk)пек (США, сухой ) (pk)пинта (британская) (pt)пинта (США, сухая) (pt)пинта (США, жидкость) (pt)кварт (британская) (qt)кварт (США, сухая) (qt)кварт (США, жидкость) (qt) столовая ложка (Can) (tbsp)столовая ложка (Imp) (tbsp)столовая ложка (U. S.) (tbsp)чайная ложка (Can) (tsp)чайная ложка (Imp) (tsp)чайная ложка (U.S.) (tsp)

    Ответ:


    Чем этот калькулятор может быть лучше?


    Получить виджет для этого калькулятора

    © Calculator Soup

    Поделитесь этим калькулятором и страницей

    Использование калькулятора

    Преобразования выполняются с использованием коэффициента преобразования. Зная коэффициент преобразования, преобразование между единицами может стать простой задачей на умножение:

    S * C = E

    Где S — наше начальное значение, C — наш коэффициент преобразования, а E — наш конечный преобразованный результат.

    Чтобы просто перевести любые единицы измерения в кубические метры , например, из 10 литров, просто умножить на значение конвертации в правом столбце в таблице ниже.

    10 л * 0,001 [(м 3 ) / (л)] = 0,01 м 3

    Чтобы преобразовать м 3 в единицы в левой колонке разделить на значение в правом столбце или умножить на обратную величину 1/x.

    0,01 м 3 / 0,001 [ (м 3 ) / (л) ] = 10 л

    Чтобы преобразовать какие-либо единицы в левом столбце, скажем, из A в B, вы можете умножить на коэффициент для A, чтобы преобразовать A в м/с 2 , затем разделить на коэффициент для B, чтобы преобразовать из м 3 . Или вы можете найти нужный вам фактор, разделив коэффициент A на коэффициент B.

    Например, чтобы перевести литры в галлоны, нужно умножить на 0,001, а затем разделить на 0,003785412. Или умножьте на 0,001/0,003785412 = 0,26417203. Таким образом, чтобы преобразовать непосредственно из л в галлоны, вы умножаете на 0,26417203.

    Единицы, символы и значения пересчета
    Используется в этом объемном калькуляторе

    Акре-футовой

    AC FT

    Кубический метр

    1233.481838

    Баррель (Империал)

    КУБИЧЕСКИЙ МЕТР

    0,163659924

    40004 40004.

    4.

    .

    40004 40004 40004 40004 40004 40004 40004 40004 40004 40004

    40004 40004 40004 40004 40004 40004 40004 40004 40004 40004 40004 40004 40004 40004 40004 40004 40004 40004 40004 40004

    40004 40004

    40004 40004

    . метр

    0,158987295

    баррель (США сухой)

    бл

    кубический метр

    0,115628199

    barrel (U.S. fluid)

    bl

    cubic meter

    0. 119240471

    bushel (Imperial)

    bu

    cubic meter

    0.03636872

    bushel (U.S. dry)

    bu

    cubic meter

    0.03523907

    шнур (дрова)

    КУРС

    Кубический метр

    3,624556364

    Кубический ног

    FT 3

    Кубический счетчик

    0,028316847

    Кабитный метр

    0,028316847

    Кабитный дюйм

    0,028316847

    .0005

    in 3

    cubic meter

    1.63871E-05

    cubic centimeter

    cm 3

    cubic meter

    0.000001

    cubic mile

    mi 3

    cubic meter

    4168181825

    Кубический двор

    YD 3

    Кубический метр

    0,764554858

    Cup (завтрак)

    C

    Кубический метр

    0,000284131

    (Кан.0005

    c

    cubic meter

    0.000227305

    cup (U.S.)

    c

    cubic meter

    0. 000236588

    ounce (Imperial fluid)

    oz

    cubic meter

    2.84131E-05

    ounce ( Флюид США)

    унции

    Кубический метр

    2,95735E-05

    Галлон (Империал)

    GAL

    Кубический метр

    0,00454609

    Галлон (США сухо0004 cubic meter

    0.004404884

    gallon (U.S. fluid)

    gal

    cubic meter

    0.003785412

    gill (Imperial)

    gi

    cubic meter

    0.000142065

    gill (U.S.)

    gi

    Кубический метр

    0,000118294

    Hogshead (Imperial)

    HHD

    Кубический метр

    0,32731848

    Hogshead (U.S.)

    HHD

    Кубический метр

    9000

    HHD

    Кубический счет0004 0.238480942

    liter

    L

    cubic meter

    0.001

    milliliter

    mL

    cubic meter

    0.000001

    peck (Imperial)

    pk

    cubic meter

    0. 00909218

    peck (U.S. dry )

    PK

    Кубический метр

    0.008809768

    Пинта (Империал)

    PT

    Кубический метр

    0,000568261

    PINT (Dry)

    0,000568261

    (США сухо0004 PT

    Кубический метр

    0,00055061

    PINT (U.S. Fluid)

    PT

    Кубический метр

    0,000473176

    (Imperial)

    QT

    .

    4000115 (Imperial)

    90005 400011136 (Imperial)

    5 400011136 (Imperial)

    5 4000115 40001136 (Imperial)

    5

    951136 (Imperial)

    591136 (Imperial)

    55

    1136 (Imperial).

    QT

    Кубический метр

    0,001101221

    Кварта (жидкость США)

    QT

    Кубический метр

    0,000946353

    Столовая ложка (канадский)

    TBSP

    0004 Кубический метр

    1.42065E-05

    Столовая ложка (Imperial)

    TBSP

    Кубический метр

    1,77582E-05

    Столовая ложка (США)

    TBSP

    .

    1 sin cos ctg: 1-Sin a*Ctg a*Cos a Помогите))))))

    2
    б)Xn=n+1/n+2…

    СРОЧНО РЕШИТЬ 1 умоляю !!!!!!!!…

    Математика

    Литература

    Алгебра

    Русский язык

    Геометрия

    Английский язык

    Химия

    Физика

    Биология

    Другие предметы

    История

    Обществознание

    Окружающий мир

    География

    Українська мова

    Українська література

    Қазақ тiлi

    Беларуская мова

    Информатика

    Экономика

    Музыка

    Право

    Французский язык

    Немецкий язык

    МХК

    ОБЖ

    Психология

    производный-калькулятор — Google Такой

    AlleBilderVideosBücherMapsNewsShopping

    suchoptionen

    Производный калькулятор • С шагами!

    www. derivative-calculator.net

    Решите производные с помощью этого бесплатного онлайн-калькулятора. Пошаговое решение и графики прилагаются!

    Переменная дифференцирования: ax_____abcdfghjklmnopqrstuvwxyz
    Сколько раз дифференцировать?: 1 2 3 4 5

    Калькулятор производных — Wolfram|Alpha

    www.wolframalpha.com › калькуляторы › производные-c…

    Бесплатный онлайн-калькулятор производных позволяет вычислять производные первого порядка и более высокого порядка, предоставляя информацию, необходимую для понимания производных …

    Калькулятор производных — Symbolab

    www.symbolab.com › Step-by-Step › Исчисление

    Бесплатный калькулятор производных — дифференцирование функций со всеми шагами. Введите любую производную функции, чтобы получить решение, шаги и график.

    Калькулятор первой производной · Калькулятор третьей производной · Калькулятор третьей производной четвертые производные, а также неявное дифференцирование и нахождение нулей/корней.

    Калькулятор производных с шагами | Калькулятор дифференциации

    calculate-derivative.com

    Калькулятор деривативов поможет вам оценить дериватив онлайн. Калькулятор дифференцирования показывает возможные промежуточные шаги, графики, корни, домен и т. д.

    Ähnliche Fragen

    Как вы рассчитываете производные?

    Есть ли калькулятор, который может вычислять производные?

    Какая производная от 2х?

    Калькулятор производных — MathPapa

    www.mathpapa.com › Калькулятор производных

    Показывает, как вычислять производные с помощью пошаговых решений! Этот калькулятор решит ваши проблемы.

    Калькулятор деривативов — Börse Frankfurt

    www.boerse-frankfurt.de › производные › деривативы-…

    Калькулятор деривативов. С помощью калькулятора деривативов вы можете проанализировать, как цена вашего дериватива будет вести себя в будущем, если цена базового …

    Пошаговый калькулятор производных — Open Omnia

    openomnia. com › калькулятор производных

    Рассчитать производная функции. Получите пошаговые решения. Попробуйте Open Omnia сегодня.

    Калькулятор производных — eMathHelp

    www.emathhelp.net › калькуляторы › исчисление-1 › производ… , цепное правило и т. д.)

    Калькулятор производных и решатель — SnapXam

    www.snapxam.com › калькуляторы › вычисление производных…

    Калькулятор производных онлайн с решением и шагами. Подробные пошаговые решения ваших задач с производными онлайн с помощью нашего математического решателя и калькулятора … 92

    Калькулятор лимита

    Калькулятор неявного дифференцирования

    China Airlines

    Управление бронированиемОнлайн-регистрацияСтатус рейса

    Забронируйте рейс

    Выберите тип поездки В одну сторону Поездка в оба конца Мульти-город и стоповер Выберите свой аэропорт

    Из Для выбора аэропортов вылета используйте клавиши со стрелками вверх и вниз.

    {{fromMessageIndex}}

    Ошибка Требуется ваша летная станция TO

    Ошибка Ваша летная станция TO недействительна

    {{fromMessageIndex}}

    Ошибка Требуется ваша летная станция TO

    Ошибка Требуется ваша летная станция TO

    К В аэропортах прибытия используйте клавиши со стрелками вверх и вниз для выбора

    {{toMessageIndex}}

    Ошибка Неверная станция полета ОТ

    Ошибка Неверная станция полета ОТ

    Ошибка Сначала выберите аэропорт вылета.

    {{toMessageIndex}}

    Ошибка Неверная станция полета ИЗ

    Ошибка Неверная станция полета ОТ

    Ошибка Сначала выберите аэропорт вылета.

    Выберите дату

    Из

    ДД/ММ/ГГГГ ДД/ММ/ГГГГ ДД/ММ/ГГГГ {{filghtsearch. date | дата:’ д МММ, гггг’}}

    Введите дату отъезда

    Введите дату

    ДД/ММ/ГГГГ ДД/ММ/ГГГГ

    tripType!=’O’ && topBarDateSelector.showInputReturn»> Введите дату возвращения

    Выберите дату отъезда.

    {{topBarDateSelector.errorMessage}}

    Выберите дату отъезда.

    Гость(и)

    {{calPassengers()}} пассажиров

    Добавить гостей

    Взрослый (12+) html’)» ng-keydown=»openStaticEnterCal($event, ‘md’, ‘myPopupTemplate.html’)» aria-popup=»true»/>

    Молодой взрослый (12-15)

    Ребенок (2-11)

    Младенец (0-2) html’)» ng-keydown=»openStaticEnterCal($event, ‘md’, ‘myPopupTemplate.html’)» aria-popup=»true»/>

    *Общее количество пассажиров не может превышать 9 человек (включая Младенца).

    К сожалению, общее количество пассажиров не может превышать 9 человек (включая младенцев).

    К сожалению, общее количество взрослых не может превышать 9 человек.

    К сожалению, общее количество молодых людей не может превышать 9 человек.

    К сожалению, общее количество детей не может превышать 8 человек.

    К сожалению, количество младенцев не может превышать количество взрослых пассажиров.

    Продолжить

    К сожалению, общее количество пассажиров не может превышать 9 человек (включая младенцев).

    К сожалению, общее количество взрослых не может превышать 9 человек.

    Извините, общее количество молодых людей не может превышать 9лиц.

    К сожалению, общее количество детей не может превышать 8 человек.

    К сожалению, количество младенцев не может превышать количество взрослых пассажиров.

    Кабина

    {{filghtsearch.travelValue}}

    • Бизнес 902:30
    • travelValue==’Premium Economy’}» ng-click=»calCloseDropCabin()»> Премиум эконом 902:30
    • Эконом 902:30

    У меня есть промокод

    Использовать корпоративный идентификатор?

    Корпоративный идентификатор

    • {{errorMsg.

    Y 1 cosx найти область определения: Найдите область определения и область значений у=1-cos x

    404 — Страница не найдена

    Страницы

    Партнеры сайта

    _________________________________


    404: Запрошенная страница с адресом [http://primer. by/algebra/funkcii/funkcija-ycosx] не найдена.

    Если Вы уверены, что набрали ссылку корректно, напишите, пожалуйста, об этом на:

    меню пользователя

    Новости


    30.11.16 


    17.03.15 


    25.03.14 


    29.08.13 


    05. 05.13 



    primer. by 2013-2016

    §3. Логико-дидактический анализ содержания темы

      1. Анализ теоретического материала

    В учебнике Алгебра и начала анализа: Учеб. для 10 кл. общеобразоват. учреждений/Ш.А.Алимов, Ю.М.Колягин, Ю.В.Сидоров и др. — М.: Просвещение, 2003. тема «Функция y=cos x, ее свойства и график» представлена в последней главе после темы «Функция y=sin x, ее свойства и график».

    Основными дидактическими единицами темы «Функция y=cos x, ее свойства и график»:

    — определения: периодическая функция

    -тригонометрические неравенства

    -функции арксинус, арккосинус, арктангенс, арккотангенс

    -свойства функций (их области определения, множества значений, четность/нечетность функций, промежутки монотонности, ограниченность, наибольшие и наименьшие значения, непрерывность и их периодичность) : y = sin x, y = cos x, y = tg x, y = ctg x, y = arcsin x, y = arccos x, y = arctg x, y = arcctg x

    — графики: y = sin x, y = cos x, y = tg x, y = ctg x, y = arcsin x, y = arccos x, y = arctg x, y = arcctg x.

    Так как начало тригонометрии, а именно тригонометрические уравнения и неравенства, изучалось в 9 классе, то в 10 вводится понятие тригонометрической функции. Способом получения новых знания может быть самостоятельное «открытие» учениками: нахождение области определения и множества значений функций y= sin x, y = cos x, y = tg x, y = ctg x; проведение доказательства четности/нечетности функции; с помощью учителя воспроизведение свойств функции y = cosx аналогичных свойствам функции y = sinx.

    Автор сначала в первом параграфе главы вводит периодичность тригонометрических функций, в нем автор вводит определение периодичной функции. Затем вводит функцию y = sin x, отмечает ее свойства и график. Потом вводится функция y =cos x на основе уже известной функции y = sinx, отмечаются ее свойства, и строится график.

    Отдельны параграф отводиться тригонометрическим неравенствам, при решении которых используются свойства тригонометрических функций, которые были пройдены, и их графики.

    После этого, необходимо отметить, что в учебнике Ш.А.Алимов в теме тригонометрические функции уделяется внимание обратным тригонометрическим функциям: y = arcsin x, y = arcos x, y = arctg x, y = arcctg x, что нет у А.Г.Мордковича, их свойствам и графикам. На них и заканчивается изучение данной главы.

    Основные цели – ввести: понятие тригонометрической функции, так как в учебнике не дается четкой формулировки понятия, но вместо определения дается словесное описание; понятие области определения и множества значений тригонометрический функций; свойств тригонометрических функций.

      1. Анализ задачного материала

    При изучение темы «Функция y=cos x, ее свойства и график» можно выделить следующие группы задач:

    1. Работа с возрастанием и убыванием функции: №959, №960, №961

    Ключевая задача №959.

    Решение:

    1)[3 ;4 ]-возрастает, 2)[-2 ;- ]-убывает, 3)[2 ; ]-убывает, 4)[ ; ]-возрастает, 5)[1;3]-убывает, 6)[-2;-1]-возрастает.

    1. Выяснить при каких значениях х, принадлежащих отрезку, функция принимает значения: №958(она же и ключевая)

    Решение:

    1)при ; 1 при 0,2 ; -1 при

    2)положительные значения при

    3)отрицательные значения при

    1. Найти все решения неравенства, принадлежащие интервалу: №962, №966

    Ключевая задача: №962

    Приведу решение одного примера: 1)cosx

    График функции y =cos x лежит не ниже графика у= при . Значит решением неравенства будет .

    1. Найти область определения функции: №963, №969-970

    Ключевая задача: №963

    1. Найти множество значений функции: №964, №968, №971

    Ключевая задача: №968

    Решение:

    1)Так как cos x убывает , то , т. е. -1

    1. Построить график: №967, №972

    Ключевая задача: №967

    Решение:

    1)у=1+cosx

    -область определения:

    -множество значений:

    -периодическая с периодом 2

    -четная

    -принимает наименьшее значение, равное 0, при , принимает наибольшее значение, равное 2, при , не отрицательная.

    -возрастает при

    убывает при

    по этим свойствам строим график.

    1. Выражая синус через косинус по формулам приведения, сравнить числа: №965(она же и ключевая).

    Решение:

    Так как убывает на и , то , т.е.

    Мэтуэй | Популярные задачи

    92
    1 Найти точное значение грех(30)
    2 Найти точное значение грех(45)
    3 Найти точное значение грех(30 градусов)
    4 Найти точное значение грех(60 градусов)
    5 Найти точное значение загар (30 градусов)
    6 Найти точное значение угловой синус(-1)
    7 Найти точное значение грех(пи/6)
    8 Найти точное значение cos(pi/4)
    9 Найти точное значение грех(45 градусов)
    10 Найти точное значение грех(пи/3)
    11 Найти точное значение арктан(-1)
    12 Найти точное значение cos(45 градусов)
    13 Найти точное значение cos(30 градусов)
    14 Найти точное значение желтовато-коричневый(60)
    15 Найти точное значение csc(45 градусов)
    16 Найти точное значение загар (60 градусов)
    17 Найти точное значение сек(30 градусов)
    18 Найти точное значение cos(60 градусов)
    19 Найти точное значение cos(150)
    20 Найти точное значение грех(60)
    21 Найти точное значение cos(pi/2)
    22 Найти точное значение загар (45 градусов)
    23 Найти точное значение arctan(- квадратный корень из 3)
    24 Найти точное значение csc(60 градусов)
    25 Найти точное значение сек(45 градусов)
    26 Найти точное значение csc(30 градусов)
    27 Найти точное значение грех(0)
    28 Найти точное значение грех(120)
    29 Найти точное значение соз(90)
    30 Преобразовать из радианов в градусы пи/3
    31 Найти точное значение желтовато-коричневый(30)
    32
    35 Преобразовать из радианов в градусы пи/6
    36 Найти точное значение детская кроватка(30 градусов)
    37 Найти точное значение арккос(-1)
    38 Найти точное значение арктан(0)
    39 Найти точное значение детская кроватка(60 градусов)
    40 Преобразование градусов в радианы 30
    41 Преобразовать из радианов в градусы (2 шт.

    Перевод из часов в дни: Перевести часы в дни онлайн ⏱️ сколько часов в сутках

    Сколько дней в 1 000 часах?

    Калькулятор «Конвертер дат»

    Сколько

    секундминутчасовднейнедельмесяцевлет

    в

    минутахчасахдняхнеделяхмесяцахгодах

    Сколько будет 1 000 часов в днях?

    Ответ: 1 000 часов это 41,67 день

    (сорок один)

    1 000 часов — это также

    • 0,114 Лет
    • или
    • 1,364 Месяц
    • или
    • 5,952 Недель
    • или
    • 41,667 День
    • или
    • 1 000 Часов
    • или
    • 60 000 Минут
    • или
    • 3 600 000 Секунд
    • или
    • 1 месяц, 13 дней и 16 часов

    1 000 часов — Отсчет времени

    Поделитесь текущим расчетом

    Печать

    https://calculat.io/ru/date/converter/days—1000—hours

    <a href=»https://calculat. io/ru/date/converter/days—1000—hours»>Сколько дней в 1 000 часах? — Calculatio</a>

    О калькуляторе «Конвертер дат»

    Онлайн калькулятор Конвертер дат поможет конвертировать Секунды/Минуты/Часы/Дни/Недели/Месяцы или года. Например, Сколько будет 1 000 часов в днях? Выберите, что Вы хотите конвертировать (например ‘дни’), количество (например ‘1000’) и целевые единицы (например ‘часы’). После этого нажмите кнопку ‘Конвертировать’.

    Калькулятор «Конвертер дат»

    Сколько

    секундминутчасовднейнедельмесяцевлет

    в

    минутахчасахдняхнеделяхмесяцахгодах

    Таблица конвертации

    Сколько дней в?Ответ (округ.)
    985 часов41,04
    986 часов41,08
    987 часов41,13
    988 часов41,17
    989 часов41,21
    990 часов41,25
    991 час41,29
    992 часа41,33
    993 часа41,38
    994 часа41,42
    995 часов41,46
    996 часов41,5
    997 часов41,54
    998 часов41,58
    999 часов41,63
    1 000 часов41,67
    1 001 час41,71
    1 002 часа41,75
    1 003 часа41,79
    1 004 часа41,83
    1 005 часов41,88
    1 006 часов41,92
    1 007 часов41,96
    1 008 часов42
    1 009 часов42,04
    1 010 часов42,08
    1 011 часов42,13
    1 012 часов42,17
    1 013 часов42,21
    1 014 часов42,25

    Когда меняются часы в Англии в 2022 и в 2023 годах?

    Часы идут назад


    30 октября

    29 марта25 октября
    28 марта31 октября
    27 марта30 октября
    26 марта29 октября

    Добавьте в свой календарь перевод часов в Великобритании (ICS, 2 КБ)

    В Великобритании часы переводятся на 1 час вперед в 01:00 в последнее воскресенье марта и на 1 час назад в 2:00 в последнее воскресенье октября.

    Период, когда часы отстают на 1 час, называется британским летним временем (BST). Вечером больше дневного света, а утром меньше (иногда это называется переходом на летнее время).

    Когда часы вернутся назад, в Великобритании будет среднее время по Гринвичу (GMT).

    Последнее обновление: 16 августа 2022 г.

    • Англия и Уэльс
    • Шотландия
    • Северная Ирландия

    Ближайший выходной день в Англии и Уэльсе29 августаЛетний банковский отпуск

    Предстоящие праздничные дни в Англии и Уэльсе

    Предстоящие праздничные дни в Англии и Уэльсе2022
    29 августа ПонедельникЛетний банковский отпуск
    26 декабря ПонедельникДень подарков
    27 декабря ВторникРождество (замещающий день)

    Предстоящие праздничные дни в Англии и Уэльсе2023
    2 января ПонедельникНовый год (замещающий день)
    7 апреля ПятницаХорошая пятница
    10 апреля ПонедельникПасхальный понедельник
    1 мая ПонедельникВыходной день в начале мая
    29 мая ПонедельникВесенний праздничный день
    28 августа ПонедельникЛетний банковский отпуск
    25 декабря Понедельник Рождество
    26 декабря ВторникДень подарков

    Если праздничный день приходится на выходной, «замещающий» будний день становится праздничным днем, обычно в следующий понедельник.

    Ваш работодатель не обязан предоставлять вам оплачиваемый отпуск в банковские или праздничные дни .

    Праздничные дни могут повлиять на то, как и когда будут выплачиваться ваши пособия .

    Добавьте в свой календарь праздничные дни в Англии и Уэльсе (ICS, 14 КБ)

    Прошедшие праздничные дни в Англии и Уэльсе 2022/2023

    Прошедшие праздничные дни в Англии и Уэльсе2022
    3 июня ПятницаПлатиновый юбилейный выходной
    2 июня ЧетвергВесенний праздничный день
    2 мая ПонедельникВыходной день в начале мая
    18 апреля ПонедельникПасхальный понедельник
    15 апреля ПятницаХорошая пятница
    3 января ПонедельникНовый год (замещающий день)

    Прошедшие праздничные дни в Англии и Уэльсе2021
    28 декабря ВторникДень подарков (запасной день)
    27 декабря ПонедельникРождество (замещающий день)
    30 августа ПонедельникЛетний банковский отпуск
    31 мая ПонедельникВесенний праздничный день
    3 мая ПонедельникВыходной день в начале мая
    5 апреля ПонедельникПасхальный понедельник
    2 апреля ПятницаХорошая пятница
    1 января ПятницаНовый год

    Прошедшие праздничные дни в Англии и Уэльсе2020
    28 декабря ПонедельникДень подарков (запасной день)
    25 декабря Пятница Рождество
    31 августа ПонедельникЛетний банковский отпуск
    25 мая ПонедельникВесенний праздничный день
    8 мая ПятницаВыходной день в начале мая (день Победы)
    13 апреля ПонедельникПасхальный понедельник
    10 апреля ПятницаХорошая пятница
    1 январяСредаНовый год

    Прошедшие праздничные дни в Англии и Уэльсе2019
    26 декабря ЧетвергДень подарков
    25 декабряСреда Рождество
    26 августа ПонедельникЛетний банковский отпуск
    27 мая ПонедельникВесенний праздничный день
    6 мая ПонедельникВыходной день в начале мая
    22 апреля ПонедельникПасхальный понедельник
    19 апреля ПятницаХорошая пятница
    1 января ВторникНовый год

    Прошедшие праздничные дни в Англии и Уэльсе2018
    26 декабряСредаДень подарков
    25 декабря Вторник Рождество
    27 августа ПонедельникЛетний банковский отпуск
    28 мая ПонедельникВесенний праздничный день
    7 мая ПонедельникВыходной день в начале мая
    2 апреля ПонедельникПасхальный понедельник
    30 марта ПятницаХорошая пятница
    1 января ПонедельникНовый год

    Прошедшие праздничные дни в Англии и Уэльсе2017
    26 декабря ВторникДень подарков
    25 декабря Понедельник Рождество
    28 августа ПонедельникЛетний банковский отпуск
    29 мая ПонедельникВесенний праздничный день
    1 мая ПонедельникВыходной день в начале мая
    17 апреля ПонедельникПасхальный понедельник
    14 апреля ПятницаХорошая пятница
    2 января ПонедельникНовый год (замещающий день)

    Последнее обновление: 16 августа 2022 г.

    Преобразование часов в дни — Преобразование времени

    Онлайн-калькулятор для преобразования часов в дни (час в д) с формулами, примерами и таблицами. Наши преобразования обеспечивают быстрый и простой способ преобразования между единицами времени.

    Калькулятор конвертации

    Введите значение в калькулятор конвертации ниже.

    Преобразование часов в дни

    СОВЕТ: Если результат вашего преобразования равен 0, попробуйте увеличить «Десятичные числа».

    Как перевести часы в дни: Введите значение в поле часов и нажмите кнопку «Рассчитать дни». Ваш ответ появится в поле дней.

    Определения преобразования

    Ниже приведен список определений, относящихся к преобразованию часов в дни.

    Что такое час (ч)?

    Час — это единица времени. Символ часа — h. Есть 24 часа в сутки.

    Что такое день (д)?

    День — это единица времени. Символ дня — d. Есть 0,0416667 дней в часе.

    Формула преобразования

    Давайте подробнее рассмотрим формулу преобразования, чтобы вы могли сами выполнить эти преобразования с помощью калькулятора или старомодного карандаша и бумаги.

    Формула для преобразования часов в дни:

     дней = часы ÷ 24 

    Пример преобразования

    Далее давайте рассмотрим пример, показывающий работу и расчеты, связанные с преобразованием часов в дни (часы в d). ).

    Преобразование часов в сутки Пример

       Задача: перевести 180 часов в дни (показать работу)  
      Формула: 
    часы ÷ 24 = дни
      Расчеты: 
    180 часов ÷ 24 = 7,5 дня
      Результат: 
    180 часов равняется 7,5 дням 

    Таблица преобразования

    Для быстрого ознакомления ниже приведена таблица преобразования, которую можно использовать для преобразования часов в дни.

    Часы в Дни.

    9 3 3 3 5 дней

    75

    часов 100074 416.6666667 days
    часов (ч) дней (г)
    1 hours 0. 0416667 days
    2 hours 0.0833333 days
    3 hours 0.125 days
    4 hours 0.1666667 days
    5 hours 0.2083333 дней
    6 часов 0,25 дней
    7 часов 0,2916667 дней
    9 hours 0.375 days
    10 hours 0.4166667 days
    20 hours 0.8333333 days
    30 hours 1.25 days
    40 hours 1.6666667 days
    50 часов 2,0833333 Days
    75 часов 3,125 дней
    100 часов 4.1666667 4.1666667 4.166667 Days 4.166667 Days0070
    250 hours 10. 4166667 days
    500 hours 20.8333333 days
    750 hours 31.25 days
    1,000 hours 41.6666667 days
    2,500 hours 104.1666667 days
    5000 часов 208,3333333 дня
    7500 часов 312,5 дней
    25,000 hours 1,041.6666667 days
    50,000 hours 2,083.3333333 days
    75,000 hours 3,125 days
    100,000 hours 4,166.6666667 days
    250,000 hours 10 416,6666667 дней

    Найти переход

    Ищете переход? Выберите тип преобразования и нужные единицы измерения.

    Поделись:

    Преобразование часов в дни

    Из

    Поменять единицы местамиСменить значок

    На

    Миллисекунды (мс)Наносекунды (нс)Секунды (с)Микросекунды (мкс)МинутыЧасыДниНеделиМесяцыГоды

    Миллисекунды (мс)Наносекунды (нс)Секунды (с)Микросекунды (мкс)МинутыЧасыДниНеделиМесяцыГоды

      Часы =   Дни

    Точность: Авто   2   3   4   5   6   7   8   9  10 12 14  16 18  20 десятичных цифр

    Преобразование часов в дни.

    Правила интегралов: способы кратко и понятно, примеры задач

    §3. Основные правила интегрирования

    I. .

    II. .

    III. Если , то.

    Неопределенный интеграл – это множество функций и равенства I и II надо понимать как совпадение множеств. Например, равенство I означает следующее: чтобы получить элементы множества , надо каждый элемент множества умножить на число .Правило III можно доказать так: . Тогда

    ,

    т.е. .

    Отметим, что правило III “работает” только тогда, когда вместо переменной интегрирования фигурирует линейная функция :

    ,

    но . Для этого интегралаправильный ответ имеет вид: .

    Замечание. Правило III есть весьма частный случай замены переменной (см. следующий параграф).

    §4. Основные методы интегрирования

    I Непосредственное интегрирование

    Так принято называть вычисление интегралов с помощью таблицы основных интегралов, правил интегрирования и тождественных преобразований подынтегральной функции.

    Примеры.

    1.

    .

    2.

    .

    Можно предложить и другой способ:

    .

    Ответы по форме различны, но формулы тригонометрии позволяют доказать их тождественность.

    3. .

    4.Частный случай формулы 14 из §2:

    .

    5.Один полезный прием:

    .

    II Метод замены переменной

    Существуют две реализации этого метода: 1) в качестве новой переменной интегрирования рассматриваем некоторую функцию , которая фигурирует в подынтегральном выражении; 2) переменную интегрированиязаменяем специально подобранной функцией.

    II.1 Подведение под знак дифференциала

    Теорема 1. Пусть известно, что . Тогда, если функция– непрерывно-дифференцируема, то

    . (1)

    Доказательство. Первое условие теоремы означает, что .

    Применяя правило дифференцирования сложной функции, получим

    ,

    что и доказывает (1).

    Чтобы воспользоваться этой теоремой на практике, необходимо вычленить в подынтегральной функции производную некоторой функции, объединить эту производную с дифференциалом переменной интегрирования и сделать замену. После вычисления интеграла вернуться к исходной переменной интегрирования.

    Примеры.

    6.

    .

    Замечание 1. Замену такого типа можно производить и без подведения под знак дифференциала.

    7.

    .

    8.

    .

    Сразу отметим здесь, что такой простой заменой не всегда удается избавить-

    ся от иррациональности (попробуйте сами, заменив в числителе на).

    Замечание 2. Приведем две формулы, которые часто встречаются и их

    лучше запомнить как табличные:

    ,

    .

    II.2 Метод подстановки

    Теорема 2. Пусть требуется вычислить интеграл и пусть– непрерывно-дифференцируемая функция, имеющая обратную. Тогда, если

    , (2)

    то

    . (3)

    Доказательство. Равенство (2) означает, что . Тогда

    .

    Сравнение начала и конца этой цепочки равенств и доказывает равенство (3).

    Пример.

    9.

    =

    .

    Ответ можно упростить, если учесть формулу синуса двойного угла:

    .

    III Интегрирование по частям

    Теорема 3. Если и– непрерывно-дифференци-руемые функции, то справедлива формула

    . (4)

    Доказательство вытекает из правила дифференцирования произведения: . Проинтегрируем обе части этого равенства и учтем одно из свойств неопределенного интеграла:

    ,

    ,

    отсюда и следует формула интегрирования по частям (3).

    При практическом применении этого метода подынтегральное выражение надо разбить в произведение таким образом, чтобы функциявычислялась просто, а интеграл в правой части (4) был бы проще исходного.

    Примеры.

    10.

    .

    .

    Замечание 3. Если при вычислении интеграла взять другую первообразную, например, получим тот же результат:

    .

    Замечание 4. Область применения этого метода в основном исчерпывается интегралами вида , где– многочлен, а– это: 1) показательные, тригонометрическиеи гипербо-лические функции; 2) логарифмические и обратные тригонометрические функции. При этом в качествев случае 1) берем многочлен, а в случае 2)– логарифмы и аркфункции. Отметим, что в случае 2) «многочлен» может содержать степени переменной с ненатуральными показателями.

    Примеры.

    12.

    .

    13.

    .

    Мы пришли к уравнению , из которого

    получаем

    .

    14.Для интеграла путем двукратного интегриро-

    вания по частям можно получить уравнение

    ,

    из которого находим

    .

    Аналогичным образом можно найти интегралы

    , , .

    3.2.2. Простейшие правила интегрирования

    Простейшие правила нахождения первообразных основаны на следующих свойствах неопределенного интеграла.

    Свойство 1. Производная от неопределенного интеграла равна подынтегральной функции

    Свойство 2. Дифференциал от неопределенного интеграла равен подынтегральному выражению. В самом деле:

    Свойство 3. Неопределенный интеграл от дифференциала функции равен самой функции плюс постоянная.

    Из второго и третьего свойств следует, что символы дифференциала и неопределенного интеграла уничтожают друг друга, будучи примененными последовательно (если отвлечься от постоянного слагаемого в последней формуле).

    Свойство 4. Интеграл от суммы конечного числа функций равен сумме интегралов от слагаемых функций:

    (3. 2.2)

    где u, υ,…,ω – функции независимой переменной х.

    Свойство 5. Постоянный множитель подынтегральной функции можно вынести за знак интеграла, т.е.:

    (3.2.3)

    где С – константа.

    Пример 4. Вычислить интеграл

    Применяя свойства интеграла, получим:

    Хотя каждое промежуточное интегрирование дает произвольное постоянное слагаемое, но их сумма снова будет произвольной постоянной.

    Теорема 2. (Об инвариантности формул интегрирования). Всякая формула интегрирования сохраняет свой вид при подстановке вместо независимой переменной любой дифференцируемой функции от нее, т.е. если

    то и

    где t = φ(x) – любая дифференцируемая функция от х.

    Доказательство. Пусть где F‘(x) = f(x).

    Возьмем теперь сложную функцию F[φ(x)] = F(t), у которой промежуточным аргументом является дифференцируемая функция t = φ(x). В силу теоремы об инвариантности (неизменности) формы первого дифференциала функции имеем:

    dF(t) = F‘(t)dt = f(t)dt

    Отсюда

    Таким образом, переменной интегрирования может быть любая функция от х. Теорема доказана.

    В силу этого правила таблица интегралов оказывается справедливой независимо от того, является ли переменная интегрирования независимой переменной или любой дифференцируемой функцией от нее.

    1.

    2.

    2а.

    2б.

    3.

    4.

    5.

    6.

    7.

    8.

    9.

    10.

    11.

    12.

    13.

    14.

    15.

    16. или

    17.

    18.

    19.

    Таблица интегралов для элементарных функций выписана в предположении, что t может быть как независимой переменной, так и любой дифференцируемой функцией от х, т.е. t = φ(x).

    Легко понять, что табличные интегралы можно было бы писать и в виде

    3. 5. 6. и т.п.

    Точно также интеграл можно записать в любом из видов Сказанное делает понятным назначение множителя dx. Он указывает на переменную интегрирования: x, t, z, u, y.

    Операция интегрирования значительно сложнее дифференцирования. Интегрирование требует индивидуального подхода к каждой функции.

    Вычислить неопределенные интегралы

    Пример 5.

    Все три интеграла табличные.

    Пример 6.

    Пример 7.

    Пример 8.

    Пример 9.

    При нахождении первообразных использованы свойства неопределенного интеграла и алгебраические преобразования подынтегральной функции, в результате все интегралы свелись к табличным.

    интеграционных правил — что такое интеграционные правила? Примеры

    Интегральные правила используются для простого вычисления интеграла. На самом деле интеграл от функции f(x) — это функция F(x) такая, что d/dx (F(x)) = f(x). Например, d/dx (x 2 ) = 2x и, следовательно, ∫ 2x dx = x 2 + C, т. е. интегрирование — это процесс, обратный дифференцированию. Но нельзя (не легко) каждый раз применять обратный процесс дифференцирования для вычисления интегралов. Правила интеграции очень помогли бы в этом отношении.

    Давайте посмотрим, каковы правила интеграции различных функций вместе с примерами.

    1. Что такое правила интеграции?
    2. Основные правила интеграции
    3. Правила интегрирования тригонометрических функций
    4. Правила интегрирования обратных тригонометрических функций
    5. Правила интеграции специальных функций
    6. Правило интеграции ILATE
    7. Правила замещения метода интеграции
    8. Правила интегрирования с использованием неполных дробей
    9. Правила интеграции FTC
    10. Часто задаваемые вопросы о правилах интеграции

    Что такое правила интеграции?

    Правила интеграции — это правила, используемые для интеграции различных типов функций. Мы видели, что ∫ 2x dx = x 2 + C, поскольку d/dx (x 2 ) = 2x. Это можно получить с помощью степенного правила интегрирования, которое гласит: ∫x n dx = x n+1 /(n+1) + C, где ‘C’ — постоянная интегрирования (которую мы добавляем после интеграла любую функцию). Используя это правило, ∫ 2x dx = 2 [x 1+1 /(1+1) ]+ C = 2 (x 2 /2) + C = x 2 + C и мы получили тот же ответ. Теперь вы, возможно, поняли важность правил интеграции. Существуют различные типы интегральных правил, и наиболее часто используемые из них перечислены ниже:

    Основные правила интеграции

    Вот основные правила интегрирования, каждое из которых может быть проверено путем дифференцирования результата. Если вы хотите увидеть, как выводится каждое из этих правил, нажмите на соответствующие ссылки.

    • Степенное правило интегрирования:
    • Интеграл от 1 равен ∫ 1 dx = x + C.
    • Интеграл от e x равен, ∫ e x dx = e x + C
    • Интеграл от x равен ∫ a x dx = a x / ln a + C
    • Интеграл от 1/x равен ∫ 1/x dx = ln |x| + С

    Кроме того, мы используем следующие свойства интегралов, когда вместо подынтегральной функции стоит сумма/разность членов.

    • ∫ [f(x)+g(x)] dx = ∫ f(x) dx + ∫ g(x) dx
    • ∫ [f(x)-g(x)] dx = ∫ f(x) dx — ∫ g(x) dx
    • ∫ a f(x) dx = ∫ f(x) dx + C, где a — константа

    Правила интегрирования тригонометрических функций

    Существует 6 тригонометрических функций: sin, cos, tan, csc, sec и cot. Вот правила интегрирования всех этих тригонометрических функций:

    • Интеграл от sin x равен ∫ sin x dx = -cos x + C.
    • Интеграл от cos x равен ∫ cos x dx = sin x + C.
    • Интеграл от tan x равен ∫ tan x = ln (sec x) + C (или) -ln |(cos x)+C
    • Интеграл csc x равен ∫ cosec x dx = ln |cosec x — cot x| + C (или) — ln |cosec x + cot x| + С (или) пер | загар (x/2) | + С
    • Интеграл от sec x равен ∫ sec x dx = ln |sec x + tan x| + C (или) (1/2) ln | (1 + sin x) / (1 — sin x) (или) ln | загар [(x/2) + (π/4)] | + С
    • Интеграл от cot x равен ∫ cot x dx = ln |sin x| + С

    Помимо этих, есть и другие правила, которые включают в себя комбинацию тригонометрических функций.

    • ∫ сек 2 x dx = tan x + C
    • ∫ cosec 2 x dx = -cot x + C
    • ∫ сек x.tan x dx = сек x + C
    • ∫ косек х . раскладушка x dx = -cosec x + C

    Правила интегрирования обратных тригонометрических функций

    Существует 6 обратных тригонометрических функций: arcsin (sin -1 ), arccos (cos -1 ), arctan (tan -1 ), arccsc (csc -1 ), arcsec (sec -1 ) и arccot ​​(cot -1 ). Вот правила интегрирования этих обратных тригонометрических функций.

    • ∫ sin -1 x dx = x sin -1 x + √(1 — x 2 ) + C
    • ∫ cos -1 x dx = x cos -1 x — √(1 — x²) + C
    • ∫ тангенс -1 x dx = x тангенс -1 x — ½ ln |1+x 2 | + С
    • ∫ csc -1 x dx = x csc -1 x + ln |x + √(x 2 — 1)| + С
    • ∫ сек -1 x dx = x сек -1 x — ln |x + √(x 2 — 1)| + С
    • ∫ раскладушка -1 x dx = x раскладушка -1 x + ½ ln |1+x 2 | + С

    На самом деле нам не нужно запоминать эти правила, вместо этого мы можем применить правило интегрирования по частям, чтобы быстро получить каждое из них.

    Помимо этих, у нас есть несколько других правил интегрирования, которые включают обратные тригонометрические функции:

    • ∫1/√(1 — x 2 ). dx = sin -1 x + C
    • ∫ 1/(1 — x 2 ).dx = -cos -1 x + C
    • ∫ 1/x√(x 2 — 1).dx = сек -1 x + C
    • ∫ 1/x√(x 2 — 1).dx = -cosec -1 x + C
    • ∫1/(1 + x 2 ).dx = тангенс -1 x + C
    • ∫ 1/(1 +x 2 ).dx = -cot -1 x + C

    Эти правила напрямую выводятся из производных обратных триггерных функций.

    Правила интеграции специальных функций

    Помимо правил, которые мы видели в предыдущих разделах, у нас есть некоторые правила интегрирования, которые используются для интегрирования некоторых специальных типов рациональных функций, где знаменатель включает квадраты. Они следующие:

    • ∫1/ (x 2 — a 2 ) dx = (1/2a) log|(x-a)/(x+a)| +С
    • ∫1/ (a 2 — x 2 ) dx = (1/2a) log|(a+x)/(a-x)| +С
    • ∫ 1/ √(x 2 + a 2 ) dx = log |x + √(x 2 + a 2 )|+C
    • ∫1/ √(x 2 — a 2 ) dx = log |x + √(x 2 — a 2 )|+C
    • ∫1/ (a 2 + x 2 ) dx = (1/a) tan -1 (х/д) + C
    • ∫ 1/ √(a 2 — x 2 ) dx = sin -1 (x/a) +C

    Существуют и другие правила интегрирования, в которых используются квадратные корни подынтегральных выражений.

    • ∫√(a 2 — x 2 ).dx = x/2 · √(a 2 — x 2 ) + a 2 /203 — 90 а + С
    • ∫√(x 2 + a 2 ).dx = x/2 · √(x 2 + a 2 ) + a 2 /2 · log |x + √(x 2 + a 2 )| + С
    • ∫√(x 2 — a 2 ).dx = x/2 · √(x 2 — a 2 ) — a 2 /2 · log |x + √(x — а 2 )| + С

    Эти 3 правила можно получить, используя метод подстановки интегрирования.

    Правило интеграции ILATE

    Правило интегрирования ILATE используется в процессе интегрирования по частям. Это применяется для интеграции произведения любых двух различных типов функций. Правило интегрирования по частям гласит:

    • ∫ у дв = ув — ∫ в дю

    Но когда у нас есть произведение функций u × dv, мы не можем понять, какая функция должна быть u, а какая — dv. В этом случае мы используем правило ILATE, где:

    • I : Обратные тригонометрические функции
    • L: Логарифмические функции
    • А: Алгебраические функции
    • T: Тригонометрические функции
    • E: экспоненциальные функции

    Первая функция «u» должна быть выбрана в соответствии с приведенным выше порядком функций с учетом первого приоритета функции, которая появляется первой в приведенном выше списке. Это правило также иногда называют LIATE. Это правило используется для интегрирования обратных тригонометрических функций (как упоминалось в одном из предыдущих разделов) и логарифмических функций. Одним из наиболее важных применений этого правила интегрирования является интеграл от ln x, то есть ∫ ln x dx = x ln x — x + C. Мы можем вывести это правило следующим образом:

    ∫ ln x dx = ∫ ln x · 1 dx

    Здесь ln x — логарифмическая функция, а 1 — алгебраическая функция. Таким образом, используя порядок ILATE, ln x должна быть первой функцией u. т. е.

    пусть u = ln x и dv = 1. Тогда

    du = (1/x) dx и v = ∫ 1 dx = x.

    По правилу интегрирования по частям dx = x ln x — ∫ 1 dx = x ln x — x + C.

    Таким образом, всякий раз, когда нет прямого правила для интегрирования функции и есть только одна функция для интегрирования, примите вторую функцию равной 1 и примените интегрирование по правилу частей.

    Правила подстановки Метод интеграции

    Когда ни одно из вышеперечисленных правил интегрирования не может быть применено, и если какая-то часть подынтегрального выражения является производной от другой части подынтегрального выражения, то используется метод подстановки. В этом методе:

    • Предположим, что часть подынтегрального выражения равна u.
    • Найти ду.
    • Полностью переведите данный интеграл через u.
    • Затем выполните интеграцию по одному из вышеуказанных правил.
    • Подставьте обратно значение u в результат.

    Пример: Найдите интеграл от ∫ 2x sin x 2 dx.

    Решение:

    Пусть x 2 = dx. Тогда 2x dx = du.

    ∫ 2x sin x 2 dx = ∫ sin u du

    = — cos u + C

    = — cos x 2 + C

    Используя этот метод подстановки, мы можем вывести несколько других правил интегрирования как следует:

    • ∫ f ‘(x) / f(x) dx = ln |f(x)| + С
    • ∫ f ‘(x) / √(f(x)) dx = 2√[f(x)] + C
    • ∫ sin ax dx = (1/a) (- cos ax) + C ;
      ∫ cos ax dx = (1/a) (sin ax) + C;
      ∫ 1/(ax + b) dx = (1/a) ln |ax + b| и т. д. (аналогичные правила можно вывести и для других функций)

    Правила интегрирования с использованием неполных дробей

    Чтобы проинтегрировать рациональную функцию, мы сначала разобьем ее на частичные дроби, используя одно из следующих правил, а затем применим правило ∫ 1/(ax + b) dx = (1/a) ln |ax + b| + C для интегрирования каждой частичной дроби. Чтобы узнать больше об интегрировании неполными дробями, нажмите здесь.

    Пример: Найдите интеграл ∫ (4x + 1) / [ (x — 2) (x + 1)] dx.

    Решение:

    Разложив приведенную выше дробь на неполные, получим: (4x + 1) / [ (x — 2) (x + 1)] = 3 / (x — 2) + 1 / ( х + 1).

    Интеграл с обеих сторон,

    ∫ (4x + 1) / [ (x — 2) (x + 1)] dx = ∫ [3 / (x — 2) + 1 / (x + 1)] dx

    Теперь применим правило ∫ 1/(ax + b) dx = (1/a) ln |ax + b| для каждой из фракций:

    ∫ (4x + 1) / [ (x — 2) (x + 1)] dx = 3 ln |x — 2| + пер |х + 1| + тел.

    Правила интеграции FTC

    FTC (Фундаментальная теорема исчисления) содержит два правила, которые помогают при интегрировании. Первое правило используется для нахождения производной неопределенных интегралов, тогда как второе правило используется для вычисления определенных интегралов.

    • FTC 1: d/dx ∫ a x f(t) dt = f(x)
    • ФТК 2: ∫ а b f(t) dt = F(b) — F(a), где F(x) = ∫ a b f(x) dx

    Пример: Найти d/dx ∫ 2 x sin t 2 dt.

    Решение:

    Здесь f(t) = sin t 2 и a = 2. По первой основной теореме исчисления имеем:

    d/dx ∫ a x 9000 t) dt = f(x)

    d/dx ∫ 2 x sin t 2 dt = f(x) = sin x 2 .

    Важные замечания по правилам интегрирования:

    • Постоянная интегрирования (C) должна добавляться к каждому результату неопределенного интеграла.
    • Постоянная интегрирования не появляется в результате определенного интеграла.
    • Примените правило LIATE для объединения произведения двух разных типов функций.
    • Для интегрирования частных функций в большинстве случаев полезен метод подстановки.

    Похожие темы:

    • Калькулятор интегралов
    • Расчетный калькулятор
    • Калькулятор производных

    Часто задаваемые вопросы о правилах интеграции

    Каковы важные правила интеграции?

    Правила интеграции — это правила, используемые для интеграции функции. Наиболее важные правила интегрирования следующие:

    • ∫ x n dx = x n+1 /(n+1) + C
    • ∫ е х dx = е х + С
    • ∫ (1/x) dx = ln |x| + С
    • ∫ a x dx = a x / ln a + C
    • ∫ 1 дх = х + С

    Что такое УФ правило интегрирования?

    Правило интегрирования UV также известно как правило интегрирования произведения (или) правило интегрирования по частям. Это правило гласит:

    ∫ u dv = uv — ∫ v du

    Здесь первая функция ‘u’ выбирается по правилу ILATE.

    Как получить правила интеграции?

    Мы знаем, что интеграция — это обратный процесс интеграции. Итак, чтобы найти интеграл функции, просто подумайте, производная от какой функции дает данную функцию. Например, чтобы вывести правило интегрирования для ∫ cos x dx, просто подумайте, «производная какой функции является cos x», тогда ответ может быть получен как sin x. Просто добавьте константу интегрирования, и тогда мы получим ∫ cos x dx = sin x + C. Однако все правила интегрирования не могут быть получены так просто. Для сложных функций вы можете обратиться ко всей этой странице.

    Что такое правило интегрирования трапеций?

    Правило интегрирования трапеций используется для нахождения приближенного значения интеграла на определенном интервале [a, b] путем деления интервала на равные n подинтервалов с конечными точками a = x 0 < x 1 < х 2 < х 3 <…..<х n = б. Правило гласит:

    b ∫ₐ f(x) dx = h/2 (f(x₀) + 2f(x₁) + 2f(x₂)) + … + f(x n )), где h = (b — a)/n.

    Что такое правило интеграции Симпсона?

    Мы используем правило интегрирования Симпсона для аппроксимации интеграла b ∫ₐ f(x) dx путем деления [a, b] на n подынтервалов, где a = x 0 < x 1 < x 2 < х 3 <…..<х н = б. Правило гласит:

    b ∫ₐ f(x) dx = h/3 (f(x₀) + 4f(x₁) + 2f(x₂)) + . .. + f(x n )), где h = (b — a)/n.

    Что такое правило интеграции средней точки?

    Используя правило средней точки интегрирования, мы можем аппроксимировать определенный интеграл b ∫ₐ f(x) dx по правилу ∑ i=1 n h f(x i * ), где h = (b — a)/n и x i * — середина интервала [x i-1 , x i ]. Здесь a = x 0 < x 1 < x 2 < x 3 <….. n = b — концы подынтервалов, когда [a, b] делится на n подынтервалы.

    Что такое правило обратной степени интегрирования?

    Степенное правило обычно относится к степенному правилу дифференцирования, которое гласит d/dx (x n ) = n x n-1 . Используя это, d/dx [x n+1 /(n+1)] = x n и, следовательно, ∫ x n dx = x n+1 /(n+1) + C. Это правило называется степенным правилом интегрирования.

    Математические слова: интегральные правила

    Математические слова: интегральные правила
    индекс: нажмите на букву
    индекс: предметные области

    Интеграл Правила

    Для следующего, a , b , c и C являются константами; для определенных интегралов они представляют действительные числовые константы. Правила применяются только тогда, когда интегралы существуют.

     

    Неопределенные интегралы (Эти все правила применяются к определенным интегралы тоже)

    1.

    2.

    3.

    4.

    5. Интегрирование по частям:

     

    Определенные интегралы

    1.

    2.

    3. Если f ( u ) ≤ г ( u ) для всех a u b , затем

    4. Если f ( u ) ≤ M для все a u b , затем

    5. Если м f ( u ) для всех a u b , затем

    6.

    Правило прямоугольника в симплекс методе: Правило прямоугольника онлайн

    Правило прямоугольника онлайн

    Правило прямоугольника применяется в методе Жордана-Гаусса.

    Алгоритм пересчета таблиц по правилу прямоугольника.
    Выбираем из старого плана четыре числа, которые расположены в вершинах прямоугольника и всегда включают разрешающий элемент РЭ.

    НЭ = СЭ — (А*В)/РЭ

    СТЭ — элемент старого плана, РЭ — разрешающий элемент, А и В — элементы старого плана, образующие прямоугольник с элементами СТЭ и РЭ.

    Назначение сервиса. Онлайн-калькулятор Правило прямоугольника предназначен для пересчета таблиц методом жордановских преобразований.

    • Шаг №1
    • Шаг №2

    Выберите размерность таблицы. 2345678910 x 2345678910

    Примечание. Данный метод не стоит путать с формулой прямоугольников.

    Пример №1. Производится пересчет элементов новой симплекс-таблицы. Каким будет значение элемента x25 в новой симплекс-таблице, если до пересчета x25 = -3 , x27 =5 , х45 = -8 , х47 =2

    Решение.
    x25 =x25 — x45*x27/x47 = -3 — (-8)*5/2 = -3+20 = 17

    Пример №2. По приведенной ниже симплекс-таблице определите, является ли соответствующее ей базисное решение оптимальным. Если решение не является оптимальным, осуществите пересчет таблицы.

      ПЧ X3 X4
    F -5 2 -1
    X1 4 2 1
    X2 3 1 2

    Решение.
    Базисное решение называется допустимым базисным решением, если значения входящих в него базисных переменных xj≥0, что эквивалентно условию неотрицательности bj≥0.
    Поскольку X1 = 4 > 0, X2 = 3 > 0, то это допустимое базисное решение. Определим, является ли оно оптимальным. Если найдется хотя бы один коэффициент индексной строки меньше нуля, то план не оптимальный, и его необходимо улучшить. В индексной строке X4 = -1 < 0, поэтому план не является оптимальным. Осуществим пересчет таблицы.

    Вычислим значения Di по строкам как частное от деления: bi / ai2 и из них выберем наименьшее:  min (4:1 , 3:2 ) = 11/2
    Следовательно, 2-ая строка является ведущей. Вместо переменной x4 в план войдет переменная x2.
    Таблица 1

      ПЧ X3 X4
    F -5 2 -1
    X1 4 2 1
    X2 3 1 2

    Разрешающий элемент РЭ=2. Строка, соответствующая переменной x2 , получена в результате деления всех элементов строки x на разрешающий элемент РЭ=2 (см. табл.2) . На месте разрешающего элемента получаем 1. В остальных клетках столбца x2  записываем нули. Все остальные элементы, включая элементы индексной строки, определяются по правилу прямоугольника. Для этого выбираем из старого плана четыре числа, которые расположены в вершинах прямоугольника и всегда включают разрешающий элемент РЭ.
    НЭ = СЭ — (А*В)/РЭ
    СТЭ — элемент старого плана, РЭ — разрешающий элемент (2), А и В — элементы старого плана, образующие прямоугольник с элементами СТЭ и РЭ (см. табл.2).
    Формируем таблицу.

    Таблица 2

    4-(3 • 1):2 2-(1 • 1):2 1-(2 • 1):2
    3 : 2 1 : 2 2 : 2
    -5-(3 • -1):2 2-(1 • -1):2 -1-(2 • -1):2

    Получаем новую таблицу:

    Таблица 3

      ПЧ X3 X2
    F -31/2 21/2 0
    X1 21/2 11/2 0
    X4 11/2 1/2 1

    Поскольку X3≥0, X2≥0, то получили оптимальный план.

    Пример №3. Решить задачу линейного программирования симплекс-методом, используя в качестве начальной угловой точки:
    f(x) = -2x1 + x2 + 4x3 – x4 – x5 → min
    x2 + 2x4 – x5 = 1
    x1 — x4 – x5 = 1
    2x2 + x3 + 2x5 = 4
    xj ≥ 0, j=1,..,5, x0 = (1;1;2;0;0)

    Решение.
    Сведем задачу F(X) → min к задаче F(X) → max. Для этого умножаем F(X) на (-1).
    0x1-1x2 + 0x3-2x4 + 1x5 = -1
    -1x1 + 0x2 + 0x3 + 1x4 + 1x5 = -1
    0x1-2x2-1x3 + 0x4-2x5 = -4
    F(x) = 2x1 — x2 — 4x3 + x4 + x5

    Затем систему ограничений преобразуем методом Гаусса-Жордана к такой форме, чтобы базисными стали переменные x1, x2, x3, а вектор b = (1, 1, 2)T


    -1
    0 -1 0 -2 1
    -1 -1 0 0 1 1
    -4 0 -2 -1 0 -2
    0 -2 1 4 -1 -1

    Итерация №1. Разрешающий элемент РЭ=-1.
    Формируем таблицу.
    Строка, соответствующая переменной x2 , получена в результате деления всех элементов строки x2 на разрешающий элемент РЭ=-1. На месте разрешающего элемента получаем 1. В остальных клетках столбца x2  записываем нули. Все остальные элементы, включая элементы индексной строки, определяются по правилу прямоугольника.
    Получаем новую таблицу:
    -1 0 -1 0 -2 1
    1 1 0 0 -1 -1
    -4 0 -2 -1 0 -2
    2 0 1 4 -3 -3

    Итерация №2. Разрешающий элемент РЭ=-1.
    Строка, соответствующая переменной x4, получена в результате деления всех элементов строки x3 на разрешающий элемент РЭ=-1. На месте разрешающего элемента получаем 1. В остальных клетках столбца x4  записываем нули.
    Все остальные элементы, включая элементы индексной строки, определяются по правилу прямоугольника.
    Получаем новую таблицу:

    -1 0 -1 0 -2 1
    1 1 0 0 -1 -1
    4 0 2 1 0 2
    -14 0 -7 0 -3 -11

    Итерация №3. Разрешающий элемент РЭ=-1. Строка, соответствующая переменной x3 , получена в результате деления всех элементов строки x1 на разрешающий элемент РЭ=-1. На месте разрешающего элемента получаем 1.  В остальных клетках столбца x3  записываем нули. Все остальные элементы, включая элементы индексной строки, определяются по правилу прямоугольника.
    Получаем новую таблицу:

    1 0 1 0 2 -1
    1 1 0 0 -1 -1
    2 0 0 1 -4 4
    -7 0 0 0 11 -18

    Далее необходимо переназначить переменные и решать симплекс-методом.

    Табличный симплекс-метод

    Для упрощения процесса решения исходные данные задачи линейного программирования при решении ее симплекс методом записываются в специальные симплекс-таблицы. Поэтому одна из модификаций симплекс метода получила название табличный симплекс метод. Задача линейного программирования в каноническом виде:
    F=a0,1x1+a0,2x2+…a0,nxn +b0 → max
    a1,1x1+a1,2x2+…a1,nxn + xn+1=b1
    a2,1x1+a2,2x2+…a2,nxn +xn+2 =b2
    …………………………………
    am,1x1+am,2x2+…am,nxn+xn+m=bm
    Исходная таблица для задачи имеет следующий вид:

    x1x2xn-1xnb
    F-a0,1-a0,2-a0,n-1-a0,n-b0
    xn+1a1,1a1,2a1,n-1a1,nb1
    xn+2a2,1a2,2a2,n-1a2,nb2
    xn+mam,1am,2am,n-1am,nbm

    x1, x2, xn – исходные переменные, xn+1, xn+2, xn+m – дополнительные переменные. Все дополнительные переменные мы приняли как базисные, а исходные переменные как небазисные (дополнительные записаны в первый столбец симплекс-таблицы а исходные в первую строку). При каждой итерации элементы симплекс-таблицы пересчитывают по определенным правилам.

      Алгоритм симплекс-метода.

     Подготовительный этап
    Приводим задачу ЛП  к каноническому виду
    F=a0,1x1+a0,2x2+…a0,nxn +b0 → max
    a1,1x1+a1,2x2+…a1,nxn+xn+1=b1
    a2,1x1+a2,2x2+…a2,nxn+xn+2=b2
    …………………………………
    am,1x1+am,2x2+…am,nxn+xn+m=bm
    В случае если в исходной задаче необходимо найти минимум – знаки коэффициентов целевой функции F меняются на противоположные a0,n=-a0,n. Знаки коэффициентов ограничивающих условий со знаком “≥” так же меняются на противоположные. В случае если условие содержит знак “≤” – коэффициенты запишутся без изменений.

     Шаг 0. Составляем симплексную таблицу, соответствующую исходной задаче

    x1x2xn-1xnb
    F-a0,1-a0,2-a0,n-1-a0,n-b0
    xn+1a1,1a1,2a1,n-1a1,nb1
    xn+2a2,1a2,2a2,n-1a2,nb2
    xn+mam,1am,2am,n-1am,nbm

     Шаг 1. Проверка на допустимость.

    Проверяем на положительность элементы столбца b (свободные члены), если среди них нет отрицательных то найдено допустимое решение (решение соответствующее одной из вершин многогранника условий) и мы переходим к шагу 2. Если в столбце свободных членов имеются отрицательные элементы то выбираем среди них максимальный по модулю – он задает ведущую строку k. В этой строке так же находим максимальный по модулю отрицательный элемент ak,l – он задает ведущий столбец – l и является ведущим элементом. Переменная, соответствующая ведущей строке исключается из базиса, переменная соответствующая ведущему столбцу включается в базис. Пересчитываем симплекс-таблицу согласно правилам.
    Если же среди свободных членов есть отрицательные элементы – а в соответствующей строке – нет то условия задачи несовместны и решений у нее нет.
    Если после перерасчета в столбце свободных членов остались отрицаетельные элементы, то переходим к первому шагу, если таких нет, то ко второму.
    Шаг 2. Проверка на оптимальность.
    На предыдущем этапе найдено допустимое решение. Проверим его на оптимальность Если среди элементов симплексной таблицы, находщихся в строке F (не беря в расчет элемент b0 – текущее значение целевой функции) нет отрицательных, то найдено оптимальное решение.
    Если в строке F есть отрицательные элементы то решение требует улучшения. Выбираем среди отрицательных элементов строки F максимальный по модулю (исключая значение функции b0)

    a0,l=min{a0,i }

    l – столбец в котором он находится будет ведущим. Для того, что бы найти ведущую строку, находим отношение соответсвующего свободного члена и элемента из ведущего столбца, при условии, что они неотрицательны.

    bk/ak,l =min {bi/ai,l } при ai,l>0, bi>0

    k – cтрока, для которой это отношение минимально – ведущая. Элемент ak,l – ведущий (разрешающий). Переменная, соответствующая ведущей строке (xk) исключается из базиса, переменная соответствующая ведущему столбцу (xl) включается в базис.

    Пересчитываем симплекс-таблицу по формулам. Если в новой таблице после перерасчета в строке F остались отрицательные элементы переходим к шагу 2

    Если невозможно найти ведущую строку, так как нет положительных элементов в ведущем столбце, то функция в области допустимых решений задачи не ограничена – алгоритм завершает работу.

    Если в строке F и в столбце свободных членов все элементы положительные, то найдено оптимальное решение.

    Правила преобразований симплексной таблицы.
    При составлении новой симплекс-таблицы в ней происходят следующие изменения:

      • Вместо базисной переменной xk записываем xl; вместо небазисной переменной xl записываем xk.
      • ведущий элемент заменяется на обратную величину ak,l‘= 1/ak,l
      • все элементы ведущего столбца (кроме ak,l) умножаются на -1/ak,l
      • все элементы ведущей строки (кроме ak,l) умножаются на 1/ak,l
      • оставшиеся элементы симплекс-таблицы преобразуются по формуле ai,j‘= ai,j– ai,lx ak,j/ ak,l

    Схему преобразования элементов симплекс-таблицы (кроме ведущей строки и ведущего столбца) называют схемой ”прямоугольника”.

    Преобразуемый элемент ai,j и соответствующие ему три сомножителя как раз и являются вершинами ”прямоугольника”.

    Пример

       

    Подробнее

    Табличные процессоры

    Реферат, Информатика

    Выполнил: EkaterinaKonstantinovna

    Решение ЗЛП симплекс-методом

    Решение задач, Информационные технологии

    Выполнил: user2202984

    Так же вы можете купить уже выполненные похожие работы. Для удобства покупки работы размещены на независимой бирже. Подробнее об условиях покупки тут.

     

    Ответ в исследовании операций для Vaishu #185201

    Решение.

    Определим максимальное значение целевой функции F(X) = 4×1 + 3×2 при следующих условиях ограничений.

    2×1 + x2≤1000

    x1 + 2×2≤800

    x1≤400

    x2≤700

    Для построения первого эталонного плана система неравенств сводится к системе уравнений путем введения дополнительных переменных.

    2х1 + х2 + х3 = 1000

    x1 + 2×2 + x4 = 800

    x1 + x5 = 400

    x2 + x6 = 700

    Матрица коэффициентов A = a (ij) этой системы уравнений имеет вид:

    A =

    2 1 1 0 0 0

    1 2 0 1 0 0

    1 0 0 0 1 0

    0 1 0 0 0 1

    Решим систему уравнений относительно основных переменных: x3, x4, x5, x6

    Предполагая, что свободные переменные равны 0, мы получаем первую базовую линию:

    X0 = (0,0,1000,800,400,700)

    Базовое решение называется допустимым, если оно неотрицательно.

    один.

    Текущий базовый план не оптимален, так как в индексной строке имеются отрицательные шансы.

    В качестве ведущего выберем столбец, соответствующий переменной x1, так как это наибольший коэффициент по модулю.

    Подсчитаем значения Di построчно как частное от деления: bi/ai1

    и выберем из них наименьшее:

    min (1000:2, 800:1, 400:1, -) = 400

    Следовательно, 3-я линия является ведущей.

    Преобразователь (1) находится на пересечении ведущего столбца и начального ряда.

    Основа B x1 x2 x3 x4 x5 x6 мин.

    x3 1000 2 1 1 0 0 0 500

    x4 800 1 2 0 1 0 0 800

    x5 400 1 0 0 0 400

    x6 700 0 10002 0 0 0 1 —

    F (X1) 0 -4 -3 0 0 0 0

    Вместо переменной x5 в план 1 будет включена переменная x1.

    Строка, соответствующая переменной x1 в плане 1, получается делением всех элементов строки x5 в плане 0 на разрешающий элемент RE = 1. Вместо разрешающего элемента получаем 1. В оставшиеся ячейки запишем нули столбец х1.

    Таким образом, в новом плане 1 заполнены строка x1 и столбец x1. Все остальные элементы нового плана 1, включая элементы индексной строки, определяются по правилу прямоугольника.

    Для этого выбираем из старого плана четыре числа, которые расположены в вершинах прямоугольника и всегда включают в себя разрешающий элемент РЭ.

    NE = SE — (A*B)/RE

    STE — элемент старого плана, RE — разрешающий элемент (1), A и B — элементы старого плана, образующие прямоугольник с элементами STE и ЧП.

    Получаем новую симплексную таблицу:

    Базис Б x1 x2 x3 x4 x5 x6

    x3 200 0 1 1 0 -2 0

    x4 400 0 2 0 1 -1 0

    x1 0 400 0 1 0

    x6 700 0 1 0 0 0 1

    F (X1) 1600 0 -3 0 0 4 0

    2.

    Текущий базовый план не является оптимальным, поскольку в индексной строке имеются отрицательные коэффициенты.

    В качестве ведущего выберем столбец, соответствующий переменной x2, так как это наибольший коэффициент по модулю.

    Вычисляем значения Di построчно как частное от деления: bi/ai2

    и выбираем наименьшее из них:

    min (200:1, 400:2, -, 700:1) = 200

    Следовательно, 1-я линия является ведущей.

    Преобразователь (1) находится на пересечении ведущего столбца и начального ряда.

    Основание B x1 x2 x3 x4 x5 x6 мин.0003

    x6 700 0 1 0 0 0 1 700

    F (X2) 1600 0 -3 0 0 4 0

    Поскольку в последнем столбце есть несколько минимальных элементов из 200, номер строки выбираем по правилу Крако. Элементы строки, имеющие одинаковое наименьшее значение min = 200, делятся на элементы предполагаемого разрешения, а результаты записываются в дополнительные строки. Для ведущей строки выбирается та, в которой наименьшее частное встречается ранее при чтении таблицы слева направо по столбцам.

    Формируем следующую часть симплексной таблицы. Вместо переменной x3 в план 2 будет включена переменная x2.

    Строка, соответствующая переменной x2 в плане 2, получается делением всех элементов строки x3 в плане 1 на разрешающий элемент RE = 1. Вместо разрешающего элемента получаем 1. В оставшиеся ячейки запишем нули столбец х2.

    Таким образом, в новом плане 2 заполнены строка x2 и столбец x2. Все остальные элементы нового плана 2, включая элементы индексной строки, определяются по правилу прямоугольника.

    • -3): 1 0- (0 • -3): 1 4 — (- 2 • -3): 1 0- (0 • -3): 1

    Получаем новую симплексную таблицу:

    Основание B x1 x2 x3 x4 x5 x6

    x2 200 0 1 1 0 -2 0

    x4 0 0 0 -2 1 3 0

    x1 400 1 0 0 0 1 0

    x6 500 0 20 -1 1

    F (X2) 2200 0 0 3 0 -2 0

    Номер итерации 2.

    1. Проверка критерия оптимальности.

    Текущий базовый план не оптимален, так как в индексной строке имеются отрицательные шансы.

    2. Определение новой базовой переменной.

    В качестве ведущего выберем столбец, соответствующий переменной x5, так как это наибольший коэффициент по модулю.

    3. Определение новой свободной переменной.

    Подсчитаем значения Di построчно как частное от деления: bi/ai5

    и выберем из них наименьшее:

    min(-, 0:3, 400:1, 500:2) = 0

    Следовательно, 2-я строка является ведущей.

    Разрешающий элемент (3) находится на пересечении ведущего столбца и начального ряда.

    Основание B x1 x2 x3 x4 x5 x6 мин

    x2 200 0 1 1 0 -2 0 —

    x4 0 0 0 -2 1 3 0 0 0 0 -1 0 2 1 250

    F (X3) 2200 0 0 3 0 -2 0

    4. Пересчет симплексной таблицы. Вместо переменной x4 план 3 будет включать переменную x5.

    Строка, соответствующая переменной x5 в плане 3, получается делением всех элементов строки x4 в плане 2 на разрешающий элемент RE = 3. Вместо разрешающего элемента получаем 1. В оставшиеся ячейки запишем нули столбец х5.

    Итак, в новом плане 3 заполнены строка x5 и столбец x5. Все остальные элементы нового плана 3, включая элементы индексной строки, определяются по правилу прямоугольника.

    B x1 x2 x3 x4 x5 x6

    200- (0 • -2): 3 0- (0 • -2): 3 1- (0 • -2): 3 1 — (- 2 • -2) : 3 0- (1 • -2): 3 -2- (3 • -2): 3 0- (0 • -2): 3

    0: 3 0: 3 0: 3 -2: 3 1: 3 3: 3 0: 3

    400- (0 • 1): 3 1- (0 • 1): 3 0- (0 • 1): 3 0 — (- 2 • 1): 3 0- (1 • 1): 3 1- (3 • 1): 3 0- (0 • 1): 3

    500- (0 • 2): 3 0- (0 • 2): 3 0- (0 • 2): 3 -1 — (- 2 • 2): 3 0- (1 • 2): 3 2 — ( 3 • 2): 3 1- (0 • 2): 3

    2200- (0 • -2): 3 0- (0 • -2): 3 0- (0 • -2): 3 3 — (- 2 • -2): 3 0- (1 • -2): 3 -2- (3 • -2): 3 0- (0 • -2): 3

    Получаем новую симплексную таблицу:

    Основание B x1 x2 x3 x4 x5 x6

    x2 200 0 1 -1/3 2/3 0 0

    x5 0 0 0 -2/3 1/3 1 0

    x1 400 1 0 2/3 — 1/3 0 0

    x6 500 0 0 1/3 -2/3 0 1

    F (X3) 2200 0 0 5/3 2/3 0 0

    1. Проверка критерия оптимальности.

    В строке индекса нет отрицательных значений. Поэтому эта таблица определяет оптимальный план задач.

    Окончательный вариант симплекс-таблицы:

    Базис Б x1 x2 x3 x4 x5 x6

    x2 200 0 1 -1/3 2/3 0 0

    x5 0 0 0 -2/3 1/3 1 0

    x1 400 1 0 2/3 -1/3 0 0

    x6 500 0 0 1/3 -2/3 0 1

    F (X4) 2200 0 0 5/3 2/3 0 0

    оптимальный план можно записать так:

    x1 = 400, x2 = 200

    F (X) = 4 * 400 + 3 * 200 = 2200.

    Ответ. 2200.

    линейное программирование — Как симплекс-метод обрабатывает тестовые коэффициенты с нулями?

    Я столкнулся с проблемой выбора опорной точки при наличии ограничений с нулевой правой стороной. Похоже, что иногда при поиске минимального коэффициента проверки следует включать нулевые тестовые коэффициенты, а иногда нет. Каково жесткое и быстрое правило для обработки нулевых тестовых коэффициентов?

    Для простой демонстрации предположим, что вы хотите максимизировать $y$ при $x + y \le 1$ и $y \le x$. График x/y пространства решений показывает треугольник с вершиной в точке $x = \frac{1}{2}, y = \frac{1}{2}$. Чтобы максимизировать $y$, мы должны оказаться в этой точке.

    Первая таблица: $$0: \begin{bматрица} & х & у & s_1 & s_2 & = \\ s_1 & 1 & 1 & 1 & 0 & 1 \\ s_2 & -1 & 1 & 0 & 1 & 0 \\ & 0 & -1 & 0 & 0 & 0 \end{bmatrix} $$

    Единственная возможная переменная — $y$ с отрицательной стоимостью -1. Затем, чтобы выбрать опорную строку, мы находим два отношения. Строка 1 $1/1 = 1$, строка 2 $0/1 = 0$. Вот в чем проблема. Как я понял, чтобы выбрать сводную строку, тест отношения должен быть положительным . Если мы будем следовать этому правилу, останется только одна возможность оставить переменную, $s_1$. Итак, попробуем следовать правилу: $$1: \begin{bматрица} & х & у & s_1 & s_2 & = \\ у и 1 и 1 и 1 и 0 и 1 \\ s_2&-2&0&-1&1&-1\ & 1 & 0 & 1 & 0 & 1 \end{bmatrix} $$

    Это нарушает еще одно правило: $s_2$ не должно быть отрицательным, верно? Во-вторых, в целевом ряду нет отрицательных значений, так что мы закончили. Но мы находимся в $x = 0, y = 1$, что даже не является решением, потому что $s_2$ является недопустимым значением. Давайте попробуем вместо этого выбрать строку 2, ту, у которой тестовый коэффициент равен 0: $$1′: \begin{bматрица} & х & у & s_1 & s_2 & = \\ s_1 & 2 & 0 & 1 & -1 & 1 \\ у и -1 и 1 и 0 и 1 и 0 \\ &-1 & 0 & 0 & 1 & 0 \end{bmatrix} $$

    Теперь нужно ввести $x$. Мы должны ориентироваться только на положительные тестовые коэффициенты. В прошлый раз мы не следовали этому правилу, но теперь будем произвольно следовать ему, и $s_1$ уйдет: $$2: \begin{bматрица} & х & у & s_1 & s_2 & = \\ x & 1 & 0 & \frac{1}{2} & -\frac{1}{2} & \frac{1}{2} \\ y & 0 & 1 & \frac{1}{2} & \frac{1}{2} & \frac{1}{2} \\ & 0 & 0 & \frac{1}{2} & \frac{1}{2} & \frac{1}{2} \end{bmatrix} $$

    И это ответ. Из любопытства, поскольку мы произвольно нарушили «положительное» правило для опорной точки $1’$ и произвольно следовали ему для опорной точки $2$, давайте попробуем быть последовательными и всегда включать ноль при поиске минимального тестового отношения.

    Пирамида ребра грани вершины ребра грани: Грани пирамиды. Вершина пирамиды. Ребра пирамиды равны.

    Грани пирамиды. Вершина пирамиды. Ребра пирамиды равны.

    • Альфашкола
    • Статьи
    • Пирамида и ее свойства


    Пирамида — это многогранник —  трехмерная геометрическая форма с плоскими гранями и прямыми ребрами. Что отличает пирамиду от других типов многогранников, так это то, что основание пирамиды является многоугольником, а все остальные грани являются треугольниками. Как видно из рисунка ниже, каждая сторона основания образует одну сторону треугольника, соединяющего основание пирамиды с ее вершиной.

     Грани имеют одну общую вершину — вершину пирамиды — и их число будет равно числу сторон, образующих основание. Пирамида с основанием, имеющим \(n\) сторон, будет иметь \(N+1\) граней т. е. один базовый многоугольник и \(N\) треугольных сторон. Многоугольник также будет иметь \(2n\) ребер и \(N + 1\) вершин. Обратите внимание, что существует особый тип пирамиды, называемый тетраэдром, для которого основание также является треугольником. Тетраэдры обладают определенными особыми свойствами, о которых мы поговорим в другой статье. В этой статье мы обсудим свойства пирамиды.

     

    Немного истории о пирамидах

    Пирамиды в Гизе, недалеко от Каира, Египет


    Различные цивилизации по всему миру за последние три тысячи лет построили пирамидальные структуры. Вероятно, самой известной и, безусловно, одной из самых больших является большая Пирамида в Гизе, на окраине Каира в Египте. Большая Пирамида, также известная как Пирамида Хуфу или Пирамида Хеопса, является самой большой и, вероятно, самой старой из нескольких пирамид на месте Гизы. Считается, что она была построена около четырех с половиной тысяч лет назад как гробница фараона Хуфу. В течение нескольких тысяч лет это было самое большое рукотворное сооружение в мире, и это единственное из оригинальных чудес древнего мира, все еще существующих. Большая Пирамида, как и многие другие египетские пирамиды, имеет квадратное основание и по форме похожа на иллюстрацию выше.

     

     

     

    Больше уроков и заданий по математике вместе с преподавателями нашей онлайн-школы «Альфа». Запишитесь на пробное занятие уже сейчас!

    Запишитесь на бесплатное тестирование знаний!

    Нажимая кнопку «Записаться» принимаю условия Пользовательского соглашения и Политики конфиденциальности

    Наши преподаватели

    Елена Анатольевна Фомина

    Репетитор по математике

    Стаж (лет)

    Образование:

    Орловский государственный университет

    Проведенных занятий:

    Форма обучения:

    Дистанционно (Скайп)

    Татьяна Сергеевна Довнар

    Репетитор по математике

    Стаж (лет)

    Образование:

    Минский Государственный Лингвистический Университет

    Проведенных занятий:

    Форма обучения:

    Дистанционно (Скайп)

    Зульашет Мовсуровна Гадаева

    Репетитор по математике

    Стаж (лет)

    Образование:

    Чеченский государственный педагогический университет

    Проведенных занятий:

    Форма обучения:

    Дистанционно (Скайп)

    Предметы

    • Математика
    • Физика
    • Химия
    • Русский язык
    • Английский язык
    • Обществознание
    • История России
    • Биология
    • География
    • Информатика

    Специализации

    • Репетитор грамматики английского языка
    • Репетитор по английскому для взрослых
    • Репетитор для подготовки к ЕГЭ по истории
    • ВПР по математике
    • Репетитор для подготовки к ВПР по английскому языку
    • ВПР по обществознанию
    • Подготовка к ЕГЭ по географии
    • Подготовка к ОГЭ по информатике
    • Программирование Pascal
    • Scratch

    Похожие статьи

    • Простые и составные числа
    • Свойства интегралов
    • Эллипс
    • Возведение обыкновенных дробей в натуральную степень
    • МИФИ: Инновационный менеджмент
    • Неравенства
    • ЕГЭ по математике, базовый уровень. Задачи на совместную работу (вариант 3)
    • Престижные премии по математике

    Нажимая кнопку «Записаться» принимаю условия Пользовательского соглашения и Политики конфиденциальности

    Текст с ошибкой:

    Расскажите, что не так

    Пирамида | matematicus.ru

    Skip to content

    Artman Стереометрия

    Пирамидой называют многогранник, у которого одна из граней является произвольным многоугольником (или треугольником или четырёхугольником и т.д. см. рис. 1), а остальные боковые грани — треугольники с общей вершиной.

    Рисунок 1

    Правильная пирамида — это пирамида, в основании которой лежит правильный многоугольник и перпендикуляр, опущенный из вершины проецируется в центр этого основания.

    Рисунок 2

    Основание пирамиды называется одна из его граней в виде произвольного многоугольника. На (рис. 2) — основанием является четырехугольник BCDE.

    Боковые грани — треугольники с общей вершиной. Как пример эта грань ADE.

    Боковые рёбра – это стороны боковых граней. В качестве примера ребро AC.

    Вершиной пирамиды называется общая вершина боковых граней.  На (рис. 2) это точка A.

    Апофема — высота боковой грани правильной пирамиды. Одна из апофем пирамиды (рис. 2) —  AP.

    Высотой пирамиды является перпендикуляр, проведённый из вершины пирамиды в центр основания, то есть AO.

    Свойства правильной пирамиды

    1. Все боковые ребра равны между собой.

    2. Все боковые грани — равные равнобедренные треугольники.

    3. Все апофемы боковых граней имеют одинаковую длину.
    4. Двухгранные углы при ребрах основания равны.

    5.  Двухгранные углы при боковых ребрах равны.

    6. Все плоские углы при вершине равны.

    7. В любую правильную пирамиду можно вписать сферу.


    Площадь полной поверхности пирамиды:

    S=Sосн.+Sб.п.

    где Sосн. — площадь основания, Sб.п. площадь боковой поверхности.

    Формула объёма пирамиды:

      

    Формула боковой площади правильной пирамиды:

     

    где φ — двухгранный угол при ребре основания пирамиды.

    Усеченная пирамида

    Усеченной пирамидой называют часть полной пирамиды, заключенная между её основанием и секущей плоскостью, параллельной основанию данной пирамиды. Боковыми гранями являются трапеции.


    Формула боковой площади правильной усеченной пирамиды:

    где p1 и p2 — периметры
    hбок. — высота боковой грани
    Формула объёма усеченной пирамиды:

    где S1 и S2 — площади оснований
    h — высота

    1072

    Пирамида имеет ___ граней, ___ ребер и ___ вершин.

    Ответ

    Проверено

    192,9 тыс.+ просмотров

    Подсказка: В этом вопросе мы должны найти общее количество граней, ребер и вершин пирамиды. Пирамида представляет собой трехмерную форму, грани которой напоминают треугольник, а также существует основание пирамиды, напоминающее форму многоугольника. Посмотрим на схему пирамиды и найдем все нужные значения граней, ребер и вершин.

    Полное пошаговое решение:
    Нам нужно найти количество граней, ребер и вершин пирамиды, которые являются ключевыми характеристиками пирамиды.
    граней: Всего у пирамиды 5$ граней. Четыре грани пирамиды представляют собой форму треугольника, а оставшаяся грань, также называемая основанием пирамиды, представляет собой форму многоугольника, и все треугольные грани всегда конгруэнтны противоположной треугольной грани. На рисунке видны $5$ грани пирамиды.
    Края: Ребра в пирамиде представляют собой соединение различных граней. Это на самом деле напоминает линию, где два лица встречаются друг с другом. Всего в пирамиде $8$ ребер. Ребра $4$ расположены на дне, который находится на стыке основания с треугольными гранями $4$, а остальные грани $4$ находятся на стыке треугольных граней. На рисунке видно, что $a,b,c,d,e,f,g$ и $h$ — ребра пирамиды.
    Вершины: вершины в пирамиде — это точки, в которых встречаются ребра. Всего в пирамиде $5$ вершин, из которых $4$ находятся в основании, а $1$ находится в точке пересечения всех вертикальных ребер. На рисунке видно, что точки $A,B,C,D$ и $E$ являются вершинами пирамиды.

    Следовательно, решение:
    Пирамида имеет 5 граней, 8 ребер и 5 вершин.

    Примечание: В этом примере мы рассмотрели различные свойства прямоугольной пирамиды, в основе которой лежит прямоугольник. Также существуют различные другие типы пирамид, такие как треугольная пирамида, квадратная пирамида, шестиугольная пирамида, пятиугольная пирамида и т. Д. Количество ребер, вершин и граней в этих типах пирамид может различаться.

    Недавно обновленные страницы

    Если ab и c единичные векторы, то влево ab2 right+bc2+ca2 математика класса 12 JEE_Main

    Стержень AB длиной 4 единицы перемещается горизонтально, когда математика класса 11 JEE_Main

    Вычислить значение intlimits0pi cos 3xdx A 0 B 1 class 12 maths JEE_Main

    Что из следующего верно 1 nleft S cup T right class 10 maths JEE_Main

    Какова площадь треугольника с вершинами Aleft class 11 maths JEE_Main

    Координаты точек A и B равны a0 и а0 класс 11 по математике JEE_Main

    Если ab и c единичные векторы, то влево ab2 right+bc2+ca2 математика класса 12 JEE_Main

    Стержень AB длиной 4 единицы перемещается горизонтально, когда математика класса 11 JEE_Main

    Вычислить значение intlimits0pi cos 3xdx A 0 B 1 class 12 maths JEE_Main

    Что из следующего верно 1 nleft S cup T right class 10 maths JEE_Main

    Какова площадь треугольника с вершинами Aleft class 11 maths JEE_Main

    Координаты точек A и B равны a0 и а0 класс 11 по математике JEE_Main

    Тенденции сомнения

    Сколько граней, ребер и вершин имеет пирамида с n-сторонним многоугольником в основании?

    Ответить

    Проверено

    225 тыс. + просмотров

    Подсказка: В этом вопросе нас спросили, сколько граней, ребер и вершин имеет пирамида.

    Чтобы решить этот вопрос, нам нужно базовое представление о гранях, ребрах и вершинах трехмерной фигуры. Давайте посмотрим на помеченную ниже диаграмму прямоугольного параллелепипеда.

    Обратите внимание, что кубоид выше имеет 6 граней, 8 ребер и 12 вершин.

    Аналогично, пирамида представляет собой трехмерную фигуру, и количество ребер, граней и вершин зависит от сторон основания.

    Более того, нам не дано какое-то конкретное основание пирамиды. Нам задали общую формулу со стороной «n».

    Полный пошаговый ответ:
    Нам задали вопрос о количестве граней, ребер и вершин пирамиды, основание которой неизвестно. Нам нужно найти обобщенную формулу в терминах «n».
    Прежде чем найти обобщенную формулу, давайте посмотрим на некоторые изображения пирамиды с разными основаниями и найдем их грани, ребра и вершины. Используя их, выведем обобщенную формулу.

    1) Грани: 
    (i) Пирамида с трехгранным основанием: Эта пирамида имеет 4 грани. Одна грань, которая является основанием, и 3 грани, соединяющие основание с вершиной (самая верхняя вершина).
    (ii) Пирамида с 4-гранным основанием: Эта пирамида имеет 5 граней. Одна грань в качестве основания и другие 4 грани, соединяющие основание с вершиной.
    Как мы видим, общее количество граней на единицу больше, чем количество сторон основания. Следовательно, пирамида с n-сторонним многоугольником в основании будет иметь $\left( {n + 1} \right)$ граней.

    2) Ребра:
    (i) Пирамида с 3-сторонним основанием: Эта пирамида имеет 6 ребер – 3 ребра составляют многоугольник в качестве основания, а остальные 3 ребра соединяют основание с вершиной.
    (ii) Пирамида с 4-сторонним основанием: эта пирамида имеет 8 ребер – 4 ребра составляют многоугольник в качестве основания, а остальные 4 ребра соединяют основание с вершиной.
    Если вы обратите внимание, общее количество ребер в два раза превышает количество сторон основания. Следовательно, пирамида с n-сторонним многоугольником в качестве основания будет иметь ($2n$) ребер.

    3) Вершины:
    (i) Пирамида с 3-сторонним основанием: Эта пирамида имеет 4 вершины – 3 вершины основания и 1 вершину.
    (ii) Пирамида с 4-гранным основанием: Эта пирамида имеет 5 вершин – 4 вершины основания и 1 вершину.
    При наблюдении мы можем сказать, что общее количество вершин на одну больше, чем количество сторон основания.
    Следовательно, пирамида с n-сторонним многоугольником в основании будет иметь $\left( {n + 1} \right)$ вершин.

    Примечание: Мы можем заметить, что пирамида представляет собой многогранник, образованный соединением многоугольного основания и точки, называемой вершиной. Каждое базовое ребро и вершина образуют треугольник, называемый боковой гранью.

    © 2015 - 2019 Муниципальное казённое общеобразовательное учреждение «Таловская средняя школа»

    Карта сайта